Sei sulla pagina 1di 167

10 seconds for aptitude

10 seconds for aptitude


2





Written to sharpen young minds who are thinkers of the future







10 seconds for aptitude



Note about the author:











10 seconds for aptitude
4


With great gratitude, I express my thanks to
U.V.C.E, B.H.S F.G.C, J.E.M.S, B.K.M.H
That provided me the platform to rise and gave me confidence. I
thank my teachers who have helped me build a personality. To
Rao sir who helped me in times of need. Thanks to my friends Ajit
Hegde, Anirudh Bhat, Deepak B, Naveen H C, Taranath Aithal,Karthik
C V, Chetan L, Shridhar NC, Pramod Aithal, Medha.S.A, Priyanka
Saraf and many others along with my first batch of dear students
who were the first ones to recognize the potential in me and gave
me hope.

10 seconds for aptitude




Preface
10 seconds is a book meant to help students understand the concept. I believe that understanding is
more important than the formula used. The best methods have been introduced to make problem
solving simple and easy. The best approach to read this book would be in the sequence given in the
book. It works out well both for an engineering student learning it overnight for placements and for
a student who wishes to learn the topics in detail. An average student can solve a moderate question
within 10 seconds if enough practice is done with the methods given. Topics that are conventionally
boring and tedious such as calendars are explained with interesting methods found by me. These
methods have been found whenever a student told me that conventional method was tedious or
while teaching in the class. Memory systems like the link and peg method are inbuilt and youll not
find it tough to remember.
The reason that this book is written is because of my students who felt the need for it.
- Sudharshan K.P.


10 seconds for aptitude
6

Contents
1) Clocks
2) Blood relations
3) Short cut techniques
4) Time and work
5) Pipes and cisterns
6) numbers
7) Calendars
8) Business mathematics
9) Permutations and combinations
10) Probability
11) Speed. Time and distance
12) Averages and allegation
13) Data analysis
14) Syllogism
15) Analytical reasoning



10 seconds for aptitude




Clocks
This topic is explained by first giving you an abstract view to solve the problems. Later explanation of how
these methods have been derived is given.
Abstract view:
The Master formula that works on most problems on clocks is as follows.
Hr*30 ~ min*
11
/
2
=
When the time is Hr:min, is the angle between hands of the clock.
In general we are asked to find at what time hands of the clock overlap, or time at which angle between
hands of clock is 90
o
or when the minute spacing is given.
Solving problems with the abstract view:
Find angle between hands of the clock at a) 4:32 b)5:36 c)6:54 d)7:37
solution: a) 4 : 32

*30 *
11
/
2

10 seconds for aptitude
8


120 176
120~176 = 56
o
that is the acute angle between hands of the clock.

b) 5 : 36

*30 *
11
/
2

150 198
The difference = 48
o
.

c) 6 : 54

*30 *
11
/
2

180 297
The difference = 117
o
.
d) ) 7 : 37

*30 *
11
/
2

210 203.5
The difference = 6.5
o
.
to multiply by 11/2, first take half the
number. Here it is 32/2=16. Next
answer = 16+16*10 = 16+160 = 176.
10 seconds for aptitude



To find time when angle between hands of the clock is given:
-} the time now is between 4 and 5 O Clock. Find time if the hands of clock are at right angles.
Solution: between 4 and 5 the time is 4:m where m is some number of minutes.
Hr*30 ~ min*
11
/
2
=
there are 2 values of time for which angle between hands of the clock is 90
o
. they are given by 2 formulae
given below
Hr*30 - min*
11
/
2
=90. 1.
min*
11
/
2
- Hr*30 =90. 2.
1 : 4*30 - m*
11
/
2
= 90
o
.
min = 5
5
/
11
.
2 : min*
11
/
2
- Hr*30 =90.
min = 38
2
/
11
.
Time is either 4:05
5
/
11
or 4:38
2
/
11
.

10 seconds for aptitude
10

Understanding the clock
A clock has a round dial with 60 divisions indicating minutes. Each space is called a minute space. After 12
hrs the hour hand returns to its original position whereas the minute hand returns after 60 minutes(1 hour.
S0, we do know that minute hand travels faster.

The Lead:-
Consider the case when time is 12 noon. After 1 hou, the hour hand would be right backat 12 travelling 60
m.s. whereas the hour hand would have travelled 5 m.s. we can say that minute hand has travelled 55 m\s
more than the hour hand. While the time elapsed is 60 minute
55 m.s is lead in 60 minutes
When the lead becomes equal to length of the track, the faster object meets the slower one
from behind. The length of circular track is 60 m.s.
55 m.s in 60 minutes
60 minutes in ? minutes

?*55 = 60*60
? =60*60/55=720/11=65
5
/
11

10 seconds for aptitude



now we found that hands of the clock overlap after every 65
5
/
11
minutes. After 12 noon, they meet
for the first time at 1:05
5
/
11

hands of the clock overlap 11 times in 12 hours.
Angle between hands of the clock.
The angle about a point is 360
o
. The minute hand covers the angle in 60 minutes so, in a minute it
covers 360/60=6
o
.the hour hand covers 360
o
in 12 hours i.e, 360/12=30
o
an hour. Since both of them move
in parallel, if 30 minutes have elapsed,so the minute hand travels 30*6=180
o
and the hour hand travels 30
degree per hour*30/60hours = 15
o
. counting from 112 noon, the time that we have considered is 12:30.
Angle between hands of the clock is 180-15=165
o
.
Angle between hands of the clock when time is Hr:Min = difference between angle covered by
minute hand and angle covered by hour hand.
Angle covered by min hand = min*6
o
.
Angle covered by hour hand = hr*30+(min/60)*30.
Net angle between hands = hr*30+(min/60)*30 ~ min*6
= hr*30+min/2 ~ min*6
= hr*30 ~ min*
11
/
2
.
10 seconds for aptitude
12

The time is between 3 O clock and 4, given angle between hands of the clock is 90
o
, find the exact
time.
Solution: between 3 and 4 the time always is 3:? (? Refers to some no of mins)
To find ?,
= 90 = hr*30 ~ min*
11
/
2
.
90 = 3*30 ~ min*
11
/
2
90 = 90 ~ min*
11
/
2
90 = 90 - min*
11
/
2
and

90 = min*
11
/
2
-90 are 2 forms that give us 2 different values of time.
min = 00 min = 32
8
/
11
are 2 values that we arrive at.
Time can be one of these 3:00 or 3: 32
8
/
11
Incorrect clocks:
An ideal clock should be such that hands of the clock overlap after every 65
5
/
11
minutes.
If the clock is such that its hands overlap earlier than 65
5
/
11
minutes then it gains time. If it takes longer
then it loses time.
10 seconds for aptitude



If a clock that was faster becomes slower than actual time it would have shown the correct time
somewhere in between.
-} a clock is 3 minutes slow at 8 am today and it was 2 minutes fast at 10 pm yesterday. When did it
show the correct time?
Solution: time elapsed = 10 hours(from 10 pm to 8 am)
Net variation in clock = 3+2 = 5 minutes
When the clock slows down by 2 minutes after 10 pm it shows correct time
Clock slows down by 5 minutes in 10 hours.
It slows down by 2 minutes in 10/5*2 = 4 hours. 4 hours from 10 pm the time will be 2 am this morning.
-} a clock overlaps after every 65 minutes. How many minutes would it gain or loose in 24 hours ?
Solution: a clock overlaps 22 times in 24 hours
Time gained a overlap is
5
/
11
minutes
Total time gained =
5
/
11
*22 = 10 minutes. The clock will be 10 minutes fast after 24 hours.
10 seconds for aptitude
14

The time in a clock is 20 minute past 2. What is the angle between the minute and hour hands of the
clock?

Given that 27th February 2003 is a Thursday. What day of the week was 27th February 1603?

integral number value?

k and then gains 2% time during the next one week. If the clock
was set right at 12 noon on a Sunday, what will be the time shown by the clock exactly 14 days from the
time it was set right?

At how many points between 10 O'clock and 11 O'clock are the minute hand and hour hand of a clock at
an angle of 30 degrees to each other?

What is the angle between the minute hand and the hour hand when the time is 1540 hours?



10 seconds for aptitude



Blood relations:

Create a rough picture of the relationship based on symbols given below standardize them and use to solve
all the problems
+ Male
- Female
Husband and wife
Generation gap
Siblings
-A +B
- C E+
F D+
The above chart is an example of a family chart written with the symbols.
A is female and grandmother of D(2 generation gaps)
10 seconds for aptitude
16

Both F and D are the grandchildren of A and B, they are the children of C and E
The relationship of F to others cannot be clearly said because; male or female is not specified. F may be the
brother of D or sister.
Only when a solution is 100% right youll mark it right.
A is the mother in law of E.
Mothers brother is the maternal uncle
Fathers brother is called paternal uncle.
Sisters son is called nephew.

The questions (1-8) are based on the following statements.
a) Seeta,Rajinder and Surinder are children of Mr. and Mrs.Maudgil
b) Renu,Raja and Sunil are children of Mr. and Mrs.Bhaskar
c) Sunil and Seeta are married and Ashok and Sanjay are their children
d) Geeta and Rakesh are children of Mr. and Mrs.Jain
e) Geeta is married to surinder and has three children named Rita,Sonu
and Raju.

1. How is Rajinder related to Raju?
a)brother b)uncle c)brother in law d)cousin
10 seconds for aptitude



e)maternal uncle

2.How is Rajinder related to Ashok?
a)brother in law b)father in law c)cousin
d)uncle
e)maternal uncle

3.How is Rakesh related to surinder?
a)brother b)cousin c)uncle d)maternal uncle
e)brother in law

4.How is rakesh related to raju?
a)brother b)cousin c)uncle d)maternal uncle
e)brother in law

5.what is sanjay's surname?
a)bhaskar b)jain c)maudgil d)surinder
e)none of these
6.Renu is sanjay's
a)sister b)sister in law c)cousin d)niece
e)aunt

7.Raju's surname is
10 seconds for aptitude
18

a)Jain b)bhaskar c)maudgil d)surinder
e)none of these

8.Sunil and Rakesh are related as
a)brothers b)cousins c)uncle and cousin d)brother in law
e)none of these


Passage (questions 9 to 12):
Amit is the son of Rahul.Sarika,Rahul's sister has a son sonu and a daughter Rita.
Raja is the maternal uncle of sonu.

9. How is Amit related to sonu?
a)Nephew b)Cousin(brother) c)uncle d)brother
e)none of these
10. How is rita related to raja?
a)sister b)daughter c)niece d)aunt
e)none of these
11.How many nephews does raja have?
a)1 b)2 c)3 d)4 e)none
12.What is the relationship of Raja with Rita?
a)uncle b)brother c)maternal uncle d)nephew
10 seconds for aptitude



e)cant be determined

Directions:Following questions pertains to Ques 13 - 15

There are six persons S1,S2,S3,S4,S5 and S6
S3 is the sister of S6
S2 is the brother of S5's husband
S4 is the father of S1 and grandfather of S6.
There are 2 fathers, one mother and 3 brothers in the family

13. Who is S5's husband?
a)S2 b)S3 c)S1 d)S4 e)S6
14.Who is the mother?
a)S1 b)S2 c)S3 d)S5 e)cannot be determined
15.How many male members are there?
a)1 b)2 c)3 d)4 d)cannot be determined

Passage (Questions 16-20)
Mr and Mrs sharma have two children Asha and Shashi. Shashi married Radha,
daughter of Mrs Mahajan. Suresh , son of Mrs Mahajan married Rita. Sonu and
Rocky are born to Suresh and Rita. Uma and Sudha are the daughters of Shashi
and Radha.

10 seconds for aptitude
20

16. What is Sudha's relation to Asha?
a) Sister b)niece c)Aunt d)Daughter
e)none of these
17.How is Sonu related to Mr Mahajan?
a)son in law b)sib c)grandson d)none of these
e)cannot be determined
18.How is Asha related to Radha?
a)mother in law b)aunt c)sister in law d)niece
e)none of the above
19.What is the surname of sonu?
a)Mahajan b)sharma c)shashi d)cannot be detemined
e)none
20.How is suresh related to sudha?
a)brother b)maternal uncle c)uncle d)cousin
e)cannot be determined

Following information pertains to ques 21-25:

-In a family of seven three generations are living together
-The family consists two married couples having two children each
-Gopal is lucky t have two grandchildren
-There are two housewives and both are beautiful
-Gopal who is Manoj's father , is a lawyer and earns the most.
10 seconds for aptitude



-jyotsna is the sister of a lecturer and herself is a nurse
-Anuradha is married to a lecturer who is Nidhi's son
-Jyothika is the grand daughter of one of the housewives and is a
classical dancer

21. What is Manoj's profession?
a)student b)lecturer c)lawyer d)cannot be determined
e)none of these
22.How many male members are there in the family?
a)2 b)3 c)4 d)cannot be determined e)none of these
23.Which of the following statements is not true?
a)The nurse is sister in law of the housewives
b)Gopal has two grand children
c)Nidhi has a son and a daughter
d)Gopal has two children
e)Anuradha has a son and a daughter
24.Who are the children of Nidhi?
a)jyotsna and manoj b)anuradha and jyotsna c)anuradha and manoj
d)cannot be determined e)none
25.who among the following is one of the married couples?
a)Gopal-jyotika b)Nidhi-Gopal c)Manoj-Jyotika d)cannot be determined
e)none
26.Anil introduces Rohit as the son of the only brother of his fathers wife.How
10 seconds for aptitude
22

is Rohit related to Anil?
a)cousin b)uncle c)brother d)father
27.Pointing out to a photograph , a man tells his friend,"she is the daughter of
the only son of my fathers wife.How is the girl in the photograph related to
the man?
a)niece b)daughter c)mother d)none of these
28.X introduces Y saying ,"He is the husband of the granddaughter of the father
of my father".How is Y related to X?
a)brother b)brother in law c)daughter d)datainadequate

Questions 29-31: I)In a family of six persons A,B,C,D,E and F there are two married
couples
II)D is grandmother of A and mother of B
III)C is wife of B and mother of F
IV)F is the grand daughter of E
now answer the following questions based on the above conditions.

29.What is C to A?
a)mother b)grandmother c)daughter d)granddaughter
30.How many male members are there in the family?
a)cannot be determined b)2 c)3 d)0
31.Who are the two couples?
a)BC and DE b)AC and DB c)cannot be determined d)none of these
10 seconds for aptitude



32.What will be the daughter of the woman who is the mother of the husband of my
mother to me?
a)mother b)aunt c)grandmother d)niece
33.A woman sees the photograph of a man and saus."this mans sister is my mother in
law". HOw is the womans husband related to the man in the photograph?
a)uncle b)brother c)father d)none of these
34.How is your mothers sisters brothers wifes child related to you?
a)sister b)brother c)cousin d)none of these

Directions(Questions 35-39):
There are six persons A,B,C,D,E and F.C is the sister of F.B is the brother
of E's husband.D is the father of A and grandfather of F.There are two fathers , three
brothers and a mother in the group.

35.Who is the mother?
a)A b)B c)D d)E
36.Who is E's husband?
a)B b)C c)A d)F
37.How many male members are there in the group?
a)one b)two c)three d)four
38.How is F related to E?
a)uncle b)husband c)son d)daughter
39.Which of the following is a group of brothers?
10 seconds for aptitude
24

a)ABF b)ABD c)BFC d)BDF
40.A party consists of grandmother,father,mother,four sons and their wives and one
son and two daughters to each of the sons.How many females are there is all?
a)14 b)26 c)18 d)none of these











10 seconds for aptitude



Short cut techniques

** a student by himself can design

shortcuts, that would be the best way to pace up. Those that are
given here are just to tell you that you can have shortcuts.**
Specialty of certain numbers:
11: multiplication by 11is fast and easy
2 3 4 5 2 3 6 2 4 5 2 7 2 1 2 3 2 3 * 11
Answer: add
2 5 7 9 7 5 9 8 6 9 7 9 9 3 3 5 5 5 3
Find a) 232456*11
b) 5342.56*11
15: multiplication by 15
Multiply the number by 10 add result to half of itself
23402*15 = 234020+1/2(234020) = 351030.
9: division by 9
10 seconds for aptitude
26

Write the first number. Add this to second number if sum does not exceed 9 write it in place of result if it
exceeds 9 add 1to the previous place and write the remainder of sum divided by 9
3 2 8 / 9
3 5
13
/
9
= 36
4
/
9
vedic method for multiplication:


multiply the numbers in units place write units place of result in the answer. 10s place is carried forward.


select 2 places next. for 3 digit by 3 digit multiplication selection of places is 1st one each, 1st two, all three, last two, last one


5 3 4 5 3 4 5 3 4 5 3 4 5 3 4
*2 3 2 2 3 2 *2 3 2 *2 3 2 *2 3 2
5*2 3*2 + 4*3 5*2+3*3+4*2 3*2+4*3 4*2
= 10 = 18 = 27 =18 = 8
answer: 1 2 0 8 8 8.

Other techniques are
base method for multiplication, common multiple methods(in this book), multiplication with fingers,
graphical methods of calculation.
But, the ones that are self designed are the best.
10 seconds for aptitude



TIME AND WORK
The concept of time and work has been a very necessary part of thinking. It is in the estimation of time
taken for a job since time unknown. From when Taj Mahal was built till todays tunnels, railway lines and
every imaginable improvement needs this estimation.
This unit will take you from the simplest examples to some of the complex problems that are faced.
-} Vikram ate 2 loaves of bread in some time. In the
same time Ram could eat 3. If they together ate a
pound in 12 min, in what time can ram alone eat
it.(pound has 10 loaves)
Did you find the answer in 10 seconds?? If not check
this out.
Soln: their capacity to eat is in the ratio
2:3::Vikram:Ram the no of loaves each one ate will also be in the same ratio so, Vikram ate 4 while Ram
ate 6. This happened in 12 minutes. Ram eats 6 loaves in 12 mins so he can eat a loaf in 2 mins and 10
loaves in 20 mins.

Conclusion: when time given is
same, work done by different
individuals is proportional to
their capacity to do work.

10 seconds for aptitude
28

Based on this conclusion we have different methods of analyzing problems. I shall introduce you to some
of them and well then follow the best.
-} Ram can do a job in 10 days. Lakshman can do it in 15 days. In how many days will they together
complete the job?
1 Per unit method:
The unit here refers to days, hours, mins depending upon the situation.
In the problem given above days is the unit.
If Ram completes 1 job in 10 days, he will do 1/10
th
of it in a day. Similarly, Lakshman will do 1/15
th
of it.
Working together, 1/10+1/15= 1/6
th
of the work is done in a day.
If 1/6
th
work is done in a day, in 6 days 1 work is done.
Drawback: dealing with fractions.

2 Relative Work method:
Name Time Capacity Relative
work (RW)
10 seconds for aptitude



Ram 10 days 3 units
30 units Lakshman 15 days 2 units
Together 6 days 5 units
Take the common multiple of 10 and 15 not necessarily the LCM but a multiple is fine.
Common multiple difference method:
Difference = 5*2=10
10 15 15*2=30
To find capacity , capacity = RW/Time.
When working together, their capacity is summed.
Time = RW/Capacity.
Note: This method is the easiest and most powerful one.




10 seconds for aptitude
30

3. Graphical Method.
10 cm

Any length 6 cm

15 cm
This method is purely of interest for readers and yet to be developed for complex problems.

We have other methods like the percentage method and direct method. But the methods introduced
are more than sufficient to solve problems that we come across.





10 seconds for aptitude



Sample problems solved : (each problem solved is different so they are to be noticed carefully.)
A and B together complete a work in 12 days, B and C together in 20 days, C and A in 15 days. In how
many days can A, B and C together complete it?
Solution:
A and B 12 5
60 B and C 20 3
C and A 15 4
A, B and C 6

Sum of units per day work of A+B + B+C + C+A = 5+3+4 =12 units =2(A+B+C)
A+B+C = 12/2 = 6 units per day.
RW = 60, time(A+B+C) = 60/6 = 10 days.
Group of people with same capacity:
The number of workers required to complete a task when all of them can work at the same rate is
explained here.
Concept: Effort input/Work output = constant.
10 seconds for aptitude
32

Classification:
Input: Number of days.
Number of hours they work per day.
Number of men working.
Output: work done in terms of length of road.
Volume of a structure.
Surface of the interior of the building. (painting)
Area of the floor.
Depending upon the question input and outputs are incorporated while solving the problem
-} 14 men built a tunnel 12 meters long in 45 days working 6 hours a day. How many men are required to
build a tunnel 60 meters long in a month if they work 7 hours a day.
Solution:
Case 1: input = 14 men, 45 days, 6 hours/day
Output = 12 meter long tunnel.
10 seconds for aptitude



Case 2: input = ? men, 30 days, 7 hours/day
Output = 60 meter long tunnel.
14*45*6/12 = ?*30*7/60
? = 90.
90 men are required for the job.

Dividend is the amount paid to workers.
When a sum of money has to be distributed among the employees, it is done based on a simple
principle that the man who works more earns more.
The concept of ratio is used here. If we have two people working together and both of them have
been working for the entire period. Then, sum of money is divided in the ratio of their capacities to work.
Rahim chopped 2 trees in a day while Azad chopped 3. After working for a week they together
received a sum of 5000 rupees. What amount should Azad get??
We know that in a day 5 trees were cut, Azad contributes to three of them. So he gets 3/5
th
of the total
money i.e. 3/5*5000=3000 rupees.
10 seconds for aptitude
34

-}15 women and 12 men together complete a job in 24 days. what is the amount a woman will get when
the total amount paid is 6200 dollars, If 3 days work of a man is done by a women in 4 days.
Solution: we will write no of men in terms of women since we know the ratio of their capacities.
3 men complete a job in a day that will take 4 women.
12 mens work will be done by 16 women (in ratio 3:4).
Total no of workers = 15 + 16 = 31 (in terms of women).
Total Amount paid is 6200 dollars for 31 women
So each women gets 6200/31 = 200 dollars.
[sum of money is divided in the ratio of their capacities to work when time is same.]
Cyclic work
We come across situations where people do not work continuously, in such cases the concept of
cyclic work will help us solve the problem.
Heena and Taj work alternately to complete a job that heena can do in 7 days and taj in 10 days, after
how many days is the job done and who finishes it?
10 seconds for aptitude



person Time capacity RW
Heena 7 10 70
Taj 10 7
Difference = 3*7/3=7
7 10 10*7=70 = RW
.
Heena can complete 10 units in a day while Taj can complete 7 units in a day. When 70 units are done the
job is complete.
We shall consider first 2 days (one days work of Heena and next days work of Taj) as one cycle. So, in a
cycle 10+7=17 units of work are done and 2 days are elapsed. Take the multiple of 17 just less than
70(nearest multiple).
17*4=68 , 68 is the multiple nearest to 70. Now 68 units are done in 4 complete cycles.
4 cycles correspond to 8 days and on the 8
th
day taj would be at work. Still we have 70-68=2 units of work
left out that Heena completes on the 9
th
day because it would be her turn.
10 seconds for aptitude
36

The exact time taken by heena to complete 2 units is 2/10 days = 1/5day. So total time taken is 8
1
/
5
days
and heena completes the job.
Ram completes 60% of a task in 15 days and then takes the help of Rahim and Rachel. Rahim is 50%
as efficient as Ram is and Rachel is 50% as efficient as Rahim is. In how many more days will they
complete the work?
Working together, A and B can do a job in 6 days. B and C can do the same job in 10 days, while C
and A can do it in 7.5 days. How long will it take if all A, B and C work together to complete the job?
Four men and three women can do a job in 6 days. When five men and six women work on the same
job, the work gets completed in 4 days. How long will a woman take to do the job, if she works alone
on it?
Shyam can do a job in 20 days, Ram in 30 days and Singhal in 60 days. If Shyam is helped by Ram
and Singhal every 3
rd
day, how long will it take for them to complete the job?
Pipe A usually fills a tank in 2 hours. On account of a leak at the bottom of the tank, it takes pipe A 30
more minutes to fill the tank. How long will the leak take to empty a full tank if pipe A is shut?
A, B and C can do a work in 5 days, 10 days and 15 days respectively. They started together to do the
work but after 2 days A and B left. C did the remaining work (in days).
X alone can do a piece of work in 15 days and Y alone can do it in 10 days. X and Y undertook to do it
for Rs. 720. With the help of Z they finished it in 5 days. How much is paid to Z?
A and B can do a piece of work in 21 and 24 days respectively. They started the work together and
after some days A leaves the work and B completes the remaining work in 9 days. After how many
days did A leave?
10 seconds for aptitude



A can complete a project in 20 days and B can complete the same project in 30 days. If A and B start
working on the project together and A quits 10 days before the project is completed, in how many days
will the project be completed?


Pipes and cisterns
A farmer opens the valve of a tank. He knows in what time it can be emptied. But if the inlet to the
tank is also open, after what time should he return to turn off the valves?
Water from the river enters a reservoir. The gates should be open when reservoir is full to its brim. If
the reservoir is half full now, after what time should the gates open? Will the reservoir overflow even
when gates are open?
Answers to these questions can be found easily. The method used is the relative value method described
while learning time and work.
Consider a simple situation where a tank has an inlet and one outlet. If the inlet alone can fill the tank in 30
minutes and outlet alone can empty it in 40 minutes, in what time will it be full when both inlet and outlet
are open.
10 seconds for aptitude
38

To solve it, we have to find a common multiple of the time given. LCM of 30 and 40 is 120. This
common multiple is considered to be a relative value of the capacity of the tank. Assume the tanks
capacity is 120 liters. With this assumption, find the rate of water flow in the inlet and outlet. Inlet fills 120
liters in 30 minutes rate of water flow is 120/30 = 4 liters per minute. Outlet empties 120 liters in 40
minutes, hence rate of flow is 120/40 = 3 liters per minute. Every minute 4 liters of water enter and 3 liters
flow out hence 1 liter remains per min. water in the tank increases at the rate of 1 liter per minute. To fill
120 liters it takes 120 minutes.
valve Time
(min)
Rate (liters
per min)
Relative
value(liters)
Inlet 30 +4
120 Outlet 40 -3
net 120 1
Inlet helps (+), outlet opposes (-), net rate is sum.

-} A hot tap fills a bucket in 30 seconds while a cold tap fills it in 20 seconds. In how many seconds will they
together fill it?
Tap
Time
seconds
Rate liters
per second
Relative
value liters
Hot inlet 30 +2 60
10 seconds for aptitude



Cold inlet 20 +3
Net 12 5


-} A bucket that should fill in 20 minutes actually takes 26 minutes to be full, due to a hole. In how many
minutes can the hole empty a full bucket?
Sol: net time is given, outlets time to be found.
Opening Time Rate Relative
value
Inlet 20 13 260
Hole 86
2
/
3
-3
Net 26 10
Net is 10 so holes rate should be -3.
-} There are 2 inlets to a tank. One can fill it in 9 minutes and the other in 18 minutes. The operator opens
both and returns at the time when tank should be full by his calculation. He finds that the outlet is open
and he closes it. It takes another 3 minutes to fill the tank. In what time can the outlet alone empty it?
Sol: operators calculation
10 seconds for aptitude
40

Time Rate Value
Inlet1 9 2
18 Inlet2 18 1
net 6 3
Operator returns after 6 minutes. But, the tank is not full. It takes 3 more minutes when outlet is closed.
From the table above we know that when the only inlets are open rate is 3. For 3 minutes they fill 9 liters.
If 9 out of 18 liters were filled in last three minutes. 9 liters were remaining after first 6 minutes. Inlets
added 18 but only 9 remained so, the outlet should have removed 9 liters in 6 minutes. The outlet can
remove 18 liters in 12 minutes.

Cyclic operation:
The concept is the same as used in time and work but both positive and negative rates are
encountered here.
-} the inlets C and D can fill a tank in 12 minutes and 14 minutes respectively. Another outlet can empty it
in 84 minutes. If they are operated in cyclic manner for 1 minute each, starting from C, which pipe will take
the last turn and time taken? Sol: first well make a table. Find relative capacity, rates this makes
calculation simpler.
Pipe Time Rate Value
10 seconds for aptitude



C 12 7
84 D 14 6
Outlet 84 1
After 1
st
minute the tank holds 7 liters
At the end of 2
nd
minute it holds 7+6 = 13 liters
Then, after 3
rd
minute it holds 13-1 = 12 liters.
12 liters are filled in a 3 minute cycle.
Take the multiple of 12 just less than 84
i.e 6
th
multiple = 12*6=72 at the end of 6 complete cycles 72 liters are filled. 6 cycles take 6*3=18 min.
next minute, C operates tank is 72+7 = 79 liters full
later D operates and tank is 79+6 = 85 already full before the minute is over.
Reanalyzing, tank is 79 liters full in 18+1 = 19 min.
Next 1 minute D can fill 6 liters but only 84-79=5 liters are required. So it takes 5/6 minutes more.
Total time taken is 19
5
/
6
minutes. D finally fills it.

10 seconds for aptitude
42

Two pipes A and B can fill a tank in 36 hours and 45 hours respectively. If both the pipes are opened
simultaneously, how much time will be taken to fill the tank?

A pipe can fill a tank in 16 hours. Due to a leak in the bottom,it is filled in 24 hours. If the tank is full,
how much time will the leak take to empty it ?

A cistern is filled by pipe A in 10 hours and the full cistern can be leaked out by an exhaust pipe B in
12 hours. If both the pipes are opened, in what time the cistern is full?

Two pipes can fill a cistern in 14 hours and 16 hours respectively. The pipes are opened
simultaneously and it is found that due to leakage in the bottom, 32 minutes extra are taken for the
cistern to be filled up. When the cistern is full in what time will the leak empty it ?

Pipes A and B can fill a tank in 20 hours and 30 hours respectively and pipe C can empty the full tank
in 40 hours. If all the pipes are opened together, how much time will be needed to make the tank full
?

Two pipes A and B can frll a tank in 24 min. and 32 min. respectively. If both the pipes are opened
simultaneously, after how much time B should be closed so that the tank is full in 18 minutes?

10 seconds for aptitude



Two pipes A and B can fill a tank in 36 min. and 4S min. respectively. A water pipe C can empty the
tank in 30 min. First A and B are opened. After 7 minutes, C is opened. In how much time, the tank Is
full ?

A tank is fitted with 8 pipes, some of them that fill the tank and others that are waste pipe meant to
empty the tank. Each of the pipes that fill the tank can fill it in 8 hours, while each of those that empty
the tank can empty it in 6 hours. If all the pipes are kept open when the tank is full, it will take exactly
6 hours for the tank to empty. How many of these are fill pipes?

Pipe A fills a tank in 30 minutes. Pipe B can fill the same tank 5 times as fast as pipe A. If both the
pipes were kept open when the tank is empty, how much time will it take for the tank to overflow?

Pipe A fills a tank of 700 litres capacity at the rate of 40 litres a minute. Another pipe B fills the same
tank at the rate of 30 litres a minute. A pipe at the bottom of the tank drains the tank at the rate of
20 litres a minute. If pipe A is kept open for a minute and then closed and pipe B is kept open for a
minute and then closed and then pipe C is kept open for a minute and then closed and the cycle
repeated, how long will it take for the empty tank to overflow?

Two pipes A and B can fill a tank in 36 hours and 45 hours respectively.If both the
pipes are opened simultaneously,how much time will be taken to fill the tank?
10 seconds for aptitude
44

a)10 hrs b)15 hrs c)20 hrs d)25 hrs
Two pipes can fill a tank in 10 hours and 12 hours respectively while a third pipe
empties the full tank in 20 hours.If all the three pipes operate simultaneously,in
how much time will the tank be filled?
a)7 hrs 30 min b)5 hrs c)7 hrs d)none of these

If two pipes function simultaneously,the reserviour will be filled in 12 hours.One
pipe fills the reservoir 10 hours faster than the other.How many hours does it take
the second pipe to fill the reservior?
a)10 b)20 c)30 d)40

A cistern has two taps which fill it in 12 minutes and 15 minutes respectively.There
is also a waste pipe in the cistern.When all the three are opened,the empty cistern
is full in 20 minutes.How long will the waste pipe take to empty the full?
a)5 min b)10 min c)15 min d)20 min

An electric pump can fill a tank in 3 hours.Because of a leak in the tank it took 3 1/2
hours to fill the tank.If the tank is full,how much time will the leak take to empty
it?
10 seconds for aptitude



a)17 b)18 c)20 d)21

Two pipes can fill a cistern in 14 hours and 16 hours respectively.The pipes are opened
simultaneously and it is found that due to leakage in the bottom it took 32 minutes more
to fill the cistern.When the cistern is full in what time will the leak empty it?
a)110 hrs b)111 hrs c)112 hrs d)none of these

Two pipes A and B can fill a tank in 36 min and 45 min resp.A water pipe C can empty
the tank in 30 min.First A and B are are opened.After 7 min,C is also opened.In how much
time,the tank is full?
a)19 min b)29 min c)39 min d)49 min

Two pipes A and B can fill a tank in 24 min and 32 min. resp.If both the pipes are
opened simultaneously,after how much time B should be closed so that the tank is full
in 18 min?
a)5 min b)8 min c)16 min d)29 min

Two pipes A and B can fill a tank in 20 and 30 min resp.If both the pipes are used
together , then how long will it take to fill the tank?
10 seconds for aptitude
46

a)12 min b)15 min c)25 min d)50 min

Three pipes A,B and C can fill a tank in 6 hours.After working at it together for 2 hrs,
C is closed and A and B can fill the remaining part in 7 hours.The number of hours
taken by C alone to fill the tank is
a)10 b)12 c)14 d)16

A cistern can be filled by a tap in 4 hours while it can be emptied by another tap in
9 hrs.If both the taps are opened simulataneously,then after how much time will the
cistern get filled?
a)4.5 hrs b)5 hrs c)6.5 hrs d)7.2 hrs

Two pipes A and B can fill a cistern in 37 1/2 minutes and 45 minutes resp.Both pipes
are opened.The cistern will be filled in just half an hour,if the pipe B is turned off
after
a)5 min b)9 min c)10 min d)15 min

A tap can fill a tank in 6 hours.After half the tank is filled,three more similar taps
are opened.What is the total time taken to fill the tank completely?
10 seconds for aptitude



a)3 hrs 15 min b)3 hrs 45 min c)4 hrs d)4 hrs 15 min

Two pipes can fill a tank in 20 and 24 min resp. and a waste pipe can empty 3 gallons
per minute.All the three pipes working together can fill the tank in 15 min.The capacity
of the tank is
a)60 gallons b)100 gallons c)120 gallons d)180 gallons

A water tank is two fifth full.Pipe A can fill a tank in 10 min and pipe B can empty it
in 6 min.If both the pipes are open,how long will it take to empty or fill the tank
completely?
a)6 min to empty b)6 min to fill c)9 min to empty d)9 min to fill

A leak in the bottom of a tank can empty the full tank in 8 hours.An inlet pipe fills
water at the rate of 6 litres a min.When the tank is full,the inlet is opened and due to
the leak,the tank is empty in 12 hrs.How many litres does the cistern hold?
a)7580 b)7960 c)8290 d)8640

Pipe A can fill a tank in 5 hrs,pipe B in 10 hrs and Pipe C in 30 hrs.If all the pipes
are open,in how many hours will the tank be filled?
10 seconds for aptitude
48

a)2 b)2.5 c)3 d)3.5

A booster pump can be used for filling as well as for emptying a tank.The capacity of
the tank is 2400 cubic meter.The emptying capacity of the tank is 10 m^3 per min higher
than its filling capacity and the pump needs 8 min lesser to empty the tank than it needs
to fill it.What is the filling capacity of the pump?
a)50 b)60 c)72 d)none of these

Pipes A and B can fill a tank in 5 and 6 hours respectively.Pipe C can empty it in 12 hrs.
If all the three pipes are opened together,then the tank will be filled in
a)1 13/17 hrs b)2 8/11 hrs c)3 9/17 hrs d)4 1/2 hrs

Three taps A,B and C can fill a tank in 12,15 and 20 hours respectively.If A is open all
the time and B and C are open for one hour each alternately,the tank will be full in
a)6 hrs b)6 2/3 hrs c)5 hrs d)7 1/2 hrs

Three pipes A,B and C can fill a tank from empty to full in 30 min,20 min and 10 min resp.
When the tank is empty,all the three pipes are opened A,B and C discharge chemical solutions
P,Q and R respectively.What is the proportion of solution R in the liquid in the tank after
3 min
10 seconds for aptitude



a)5/11 b)6/11 c)7/11 d)8/11

Two pipes A and B can fill a tank in 6 hrs and 4 hrs resp.If they are opened on alternate
hours and if pipe A is opened first,in how many hours,the tank shall be full?
a)4 b)4 1/2 c)5 d)5 1/2

Two pipes A and B can separately fill a cistern in 60 min and 75 min respectively.There is
a third pipe in the bottom of the cistern to empty it.If all the three pipes are simultaneously
opened,then the cistern is full in 50 min.In how much time,the third pipe alone can empty
the cistern?
a)90 min b)100 min c)110 min d)120 min

Two pipes A and B can fill a cistern in 12 min and 15 resp,while a third pipe C can empty
the full tank in 6 min.A and B are kept open for 5 min in the beginning and then C is also
opened.In what time the cistern is emptied?
a)30 min b)33 min c)37 1/2 min d)45 min

A pump can fill a tank with water in 2 hrs.Because of a leak,it took 2 1/3 hours to fill the
tank.The leak can drain all the water of the tank in
a)4 1/3 hrs b)7 hrs c)8 hrs d)14 hrs
10 seconds for aptitude
50

A large tanker can be filled by two pipes A and B in 60 min and 40 min resp.How many min will
it take to fill the tanker from empty state if B is used for half the time and A and B fill
it together for another half?
a)15 min b)20 min c)27.5 min d)30 min

Two taps A and B can fill a tank in 5 hrs and 20 hrs resp.If both the taps are open then due
to leakage,it took 30 min more to fill the tank.If the tank is full,how long will it take
for the leakage alone to empty the tank?
a)4 1/2hrs b)9 hrs c)18 hrs d)36 hrs

Two pipes A and B can fill a tank in 15 hrs and 20 hrs resp while a third pipe C can empty
the full tank in 25 hrs.All the three pipes are opened in the beginning.After 10 hours,C is
closed.In how much time will the tank be full?
a)12 hrs b)13 hrs c)16 hrs d)18 hrs
Two pipes A and B together can fill a cistern in 4 hours.Had they been opened separately,then
B would have taken 6 hours more than A to fill the cistern.How much time will be taken by A
to fill the cistern separately?
a)1 hr b)2 hrs c)6 hrs d)9 hrs

10 seconds for aptitude



Two pipes A and B can fill a tank in 15 min and 20 min resp.Both the pipes are opened together
but after 4 min,pipe A is turned off.What is the total time required to fill the tank?
a)10 min 20 sec b)11 min 45 sec c)12 min 30 sec d)14 min 40 sec
One pipe can fill a tank three times as fast as another pipe.If together the two pipes can
fill the tank in 36 min,then the slower pipe alone will be able to fill the tank in
a)81 min b)108 min c)144 min d)193 min









10 seconds for aptitude
52

Numbers
Real complex
Rational irrational
Integers non terminating,
Decimal numbers non recurring
Terminating numbers
Recurring decimals

Integers

Negative non negative positive
(-1, -2, .. -) (0,1,2,..) (1,2,3,.)
Whole numbers Natural numbers

10 seconds for aptitude



Natural numbers
Even(E) Odd(O) Prime Composite
E+E = E E*E = E E/E = E
E+O = O E*O = E E/O = E
O+O = E O*O= O O/O = O
O/E = Decimal
Prime factorization:
N = P1
a
* P2
b
* P3
c

Number of factors of N = (a+1)(b+1)(c+1)
Ex: 150 = 3*5*5*2
= 3
1
* 5
2
* 2
1

Number of factors = (1+1)(2+1)(1+1) = 12.


10 seconds for aptitude
54

Highest common factor (greatest common multiple)
Method 1: long division method
2 80 140
2 40 70 HCF = 2*2*5 = 20
5 20 35 N1 = HCF*C1 = 20*4 = 80
4 7 N2 = HCF*C2 = 20*7 = 140
C1 C2

Method 2: prime factorization method
80 = 2*2*2*2*5
= 2
4
* 5 TAKE THE COMMON FACTORS
140 = 2*2*5*7 WITH THEIR LOWEST POWERS
= 2
2
* 5 * 7 AND MULTIPLY THEM
HCF = 2
2
* 5 = 20
10 seconds for aptitude



Method 3: simple division method
80 140 1
-80
60 80 1
-60
20 60 3
-60
00
HCF = 20
KPs method:
Difference= 60 *
80
/
60
= 80
80 140
To get back
80
/
60
=
4
/
3
HCF=
60
/
3
= 20
10 seconds for aptitude
56

Lowest common multiple:
Method 1: long division method
2 80 140
2 40 70 LCM = 2*2*5*4*7 = 560
5 20 35 N1 = LCM/C2 = 560/7=80
4 7 N2 = LCM/C1 =560/4 =140
C1 C2

Method 2: prime factorization method
80 = 2*2*2*2*5
= 2
4
* 5 TAKE ALL THE FACTORS
140 = 2*2*5*7 WITH THEIR HIGHEST POWERS
= 2
2
* 5 * 7 AND MULTIPLY THEM
LCM = 2
4
* 5 * 7 = 560
10 seconds for aptitude



KPs method:
Difference= 60 *
80
/
60
= 80
80 140
To get back
80
/
60
=
4
/
3
HCF=
60
/
3
= 20
LCM = 140*4 = 560
Sum of numbers N1+N2 = HCF*(C1+C2)
Difference N1-N2 = HCF*(C1-C2)
Product N1*N2 = HCF*LCM
For fractions:
LCM =
LCM of numerators
/

HCF of denominators
HCF =
HCF of numerators
/

LCM of denominators

10 seconds for aptitude
58

Remainders
If N/a gives remainder R
And N/b gives the same remainder R
N = k * ( LCM(a, b) ) + R
k = 1,2,3, least value is obtained when k=1

Divisibility rules:

To be divisible by
2 => last digit should be even
3 => sum of digits should be divisible by 3
4 => last 2 digits divisible by 4 or 00
5 => last digit should be 0 or 5
6 => it should be divisible by 2 and 3

10 seconds for aptitude



7 => method 1
Ex:- 343 3 4 3
- 6 * 2
2 8
28 is divisible by 7 so 343 is also divisible.
Method 2
(1/7)
r
= 1, (10/7)
r
= 3, (100/7)
r
= 2, (1000/7)
r
= -1
(10000/7)
r
= -3 the same sequence follows that is
2 3 1 -2 -3 -1 2 3 1.
Ex:- 563829
The numbers along with place values are written.
5 6 3 8 2 9 result should be
*-2 *-3 *-1 *2 *3 *1 0 or 7
-10 -18 -3 +16` +6 +9 = 0
10 seconds for aptitude
60

=>8 last 3 digits should be divisible by 8 or 000
=> 9 sum of digits should be divisible by 9
=> 10 last digit 0
=> 11 sum of digits at odd places sum of digits at even places should be 0 or multiple of 11.
Ex:- 353, -3 +5 -3 = 1 it is not divisible
352, -3 +5 -2 = 0 it is divisible
=>12 should be divisible by both 4 and 3
For any number, its factors who are a prime composite pair should be taken
N is divisible by X, if X is divisible by both prime composite factors
Prime composite factors: pair of factors of the number who have only one common factor 1.

Power cycles:
Find units place of 2222
2435
* 893
456
?
Sol: only last digit of answer is written in table below
10 seconds for aptitude



X
1
X
2
X
3
X
4
X
5
frequency
1 1 1 1 1 1
2 4 8 6 2 4
3 9 7 1 3 4
4 6 4 6 4 2
5 5 5 5 5 1
6 6 6 6 6 1
7 9 3 1 7 4
8 4 2 6 8 4
9 1 9 1 9 2

We can observe from the table above that units place of the result repeats again.
This happens in a cyclic manner
The number of powers after which the number x occurs in units place is the frequency
For any number, to find the power cycle you need not write the whole table
Cycle of that particular number is enough.
If the 3
rd
power of 8 ends with 2, 8*2 = 16 ends with 6 so 4
th
power ends with 6, finding the units place
value of power is very easy
10 seconds for aptitude
62

2222
4
to find units place for this, from cycle of 2, that is 2 4 8 6 we know that 4
th
place
corresponds to 6. Hence the result will end with 6.
2222
56
to find units place for this, from cycle of 2 we know that its frequency is 4. Find remainder of the
division (56/4)
r
= 0 if the remainder is 0 when divided by 4 it is same as remainder 4.
So 4
th
place in cycle is 6, the result will end with 6
The question asks us to find units place of 2222
2435
* 893
456
2222
2435
will end with?
Cycle of 2 has frequency 4,
(2435/4)
r
= from divisibility rules we know that for a number to be divisible by 4, its last two digits should
be divisible by 4. Remainder obtained by dividing last 2 digits is the same as remainder of whole division.
(35/4)
r
=3 , (2435/4)
r
=3
So third place in the cycle of 2 = 8.
893
456
will end with?
Units place is 3, cycle of 3 has frequency 4
(456/4)
r
=0. Hence the 4
th
place in cycle of 3 = 1.
10 seconds for aptitude



8 * 1 will end with 8.
Hence the above calculation ends with 8.
Finding remainders for numbers with powers
Find the remainder when 3
75
is divided by 4?
Sol:
express the numerator in terms of the denominator.
3 = 4-1 hence (3
75
/4)
r
= ((4-1)
75
/4)
r
From binomial theorem, we know that in the result of the expansion (4-1)
75
all the terms will have 4 except
the last term. So, all the terms except the last one is a multiple of 4 so, obviously they will not leave any
remainder when divided by 4. Proceed with the remaining
((-1)
75
/4)
r
= -1 remainder is -1. It is the same as 4-1 =3.



10 seconds for aptitude
64

Progressions
Arithmetic progression:
a, a+d, a+2d, a+3d. a+(n-1)d (n
th
term)
a = 1
st
term, d = common difference, n = number of terms
n
th
term , T
n
= a + (n-1)d.
summation, S
n
=
n
/
2
(1
st
term + last term)
S
n
=
n
/
2
(a + a+(n-1)d)
Arithmetic mean = (a+b+c)/3.
Geometric progression:
a, ar, ar
2
, ar
3
,a
4
, . ar
n-1

a = 1
st
term, r = common ratio, n = number of terms, T
n
= ar
n-1
.
S
n
= a(r
n
- 1)/(r-1) for r>1
S
n
= a(1-r
n
)/1-r for r <1
S
n
= a/(1-r) for r<1 as r
10 seconds for aptitude



Geometric mean = 2
Ex:- A ball is dropped from a height of 10 meter. Every time it bounces back to 3/5
th
of its previous height.
What is the distance travelled by the ball before coming to rest.
Sol: Distances travelled are 10m, 10
3
/
5
m, 10(
3
/
5
)
2
m.. and so on till 0
a = 10, r =
3
/
5
, n = 1,2,3...
S
n
as n
S
n
= a/(1-r) = 10/(1-
3
/
5
) = 25m downwards,
Upwards, it is 25-10 = 15m upwards,
Total distance travelled = 25+15 = 40m.





10 seconds for aptitude
66

Ex:- find total length of lines in the figure.



20 cm
Sol:
10 cm
10 cm
Length of hypotenuse from Pythagoras theorem =

= 10*2
1/2

= 20/
Perimeter = 4( 20 + 20/ + 20/)
2
+.)
= 4 S
n
= 4 * ( 20/(1-1/) ) = 80

.
1.Find the number of factors that the number 2025 has.
a)12 b)13 c)15 d)14
10 seconds for aptitude



2.In how many ways can 14630 be writen as the product of two factors
a)14 b)15 c)23 d)16
3.Find the largest 3 digit multiple of 28.
a)980 b)987 c)986 d)578
4.Find the smallest number which when divided by 4 or 7 leaves a
remainder of 3 in each case and the number being greater than the
two divisors
a)30 b)76 C)21 d)31
5.Find the largest four digit number which when divided by 5 and 9
gives remainders of 3 and 7 respectively
a)8899 b)9988 c)9898 d)8989
6.Find the smallest number which when divided by 7 leaves a remainder
of 6 and when divided by 11 leaves a remainder of 8
a)40 b)20 c)59 d)41
7.Find the remainder of the division 3^42/4
a)0 b)1 c)2 d)3
8.Which of the following is divisible by 19?
a)614120 b)860472 c)921194 d)998777
9.What should be added to 475935 so that it becomes a multiple of 11?
a)2 b)3 c)7 d)8
10 seconds for aptitude
68

10.What should be subracted from 478185 so that it becomes a multiple
of 19?
a)7 b)8 c)10 d)12
11.There are some sweets with me.If i distribute them equally among 10,
16 or 20 children,i would be left with one sweet in each case.If i
distribute the sweets equally among 20 children,I do not have any
sweets left with me.How many sweets do i have?
a)92 b)115 c)150 d)161
12.What is the largest number which,divides 288,528 and 708, and leave
the same remainder in each case?
a)30 b)60 c)75 d)150
13.What is the smallest five digit number which when divided by 7,11 and
21 leaves a reminder of 5 in each case?
a)10164 b)10169 c)10118 d)10123
14.From a certain city,buses start for four different places every 15,20,
25 and 30 minutes starting from 8 a.m.At what time,for the first time
after 8 a.m. would all the buses start together again?
a)10 a.m b)12 noon c)1 p.m d)2 p.m
15.A number when divided by 928 leaves a reminder 244.What would be the
reminder when the number is divided by 58?
10 seconds for aptitude



a)8 b)12 c)17 d)23
16.When a three digit number is divided by 64484 and 62767,the remainder
is the same in both the cases.What is the remainder?
a)101 b)458 c)757 d)cannot be determined
17.if x+1/x = 4 , find the value of x^4-1/x^4(x>1)
a)112 b)112sqrt3 c)224 d)none of these
18.Which of the following is a perfect square?
a)20,734 b)61,504 c)71,295 d)77,286
19.How many digits are there in the smallest number containing all 9's
which is divisible by 13?
a)4 b)5 c)6 d)8
20.If the number 7448x24y is divisible by 72,find the value of x-y given
x not equal to y
a)5 b)7 c)8 d)none of these
21.The difference of a four digit number and any number formed by permutin
its digits would always be divisible by
a)18 b)11 c)10 d)9
22.Find a number such that it exceeds 18 by three times the number by which
it is less than 86
a)32 b)69 c)54 d)67
10 seconds for aptitude
70

23.Find the smallest number,which when divided by 31 leaves as remainder 7
and when divided by 25 leaves a remainder 6
a)106 b)81 c)131 d)162
24.Find the greatest number with which when 565,847 and 1551 are divided,
the respective remainders are 5,7 and 11
a)70 b)140 c)170 d)210
25.Find the remainder when 3^50 is divided by 11
a)1 b)6 c)7 d)cannot be determined
26.Find the smallest number which when increased by 5 is divisible by 36,
108,126 and 198
a)4422 b)8311 c)7764 d)6867
27.What is the minimum number of identical square tiles required to cover
a floor of dimensions 3 m 78 cm by 4 m 80 cm
a)3200 b)5040 c)7600 d)8100
28.The HCF and LCM of a pair of numbers are 11 and 1001 resp.Find the smaller
of the two numbers given that their sum is 220
a)11 b)33 c)55 d)77
29.Find the greatest power of 20,which can divide 200!
a)10 b)49 c)98 d)104
30.Find the number of factors of the number 46200
10 seconds for aptitude



a)64 b)78 c)96 d)106
31.Find the highest power of 5 contained in 250!
a)50 b)62 c)78 d)84
32.Find the largest five digit number which when divided by 8 leaves remainder 3
and when divided by 7 leaves remainder 1?
a)99948 b)99953 c)99960 d)99947
33.When the numbers 5,7 and 11 divide a multiple of 17,the remainders left are
,respectively,4,6 and 10.Which multiple of 17 gives the least number that
satisfies the given condition?
a)384 b)317 c)385 d)none of these
34.How many times the HCF , of the fractions 3/14,6/35 and 16/21 is their LCM?
a)1440 b)643144 c)210 d)48
35.What is the maximum number of ways in which the number 11025 can be expressed
as the product of a pair of co prime factors
a)1 b)2 c)4 d)8
36.What is the number of different ways in which the number 784 can be expressed
as a product of the two different factors?
a)8 b)7 c)2 d)none of these
37.What is the sum of all the coprimes of 24,which are less than 24?
a)96 b)72 c)52 d)144
10 seconds for aptitude
72

38.How many co prime of 53 are there between 29 and 41,including the two numbers?
a)23 b)13 c)36 d)none of these
39.What is the HCF of the numbers represented by n(n^2 + 20), n being an even
number?
a)16 b)12 c)8 d)48
40.When a three digit number is divided by 64484 and 62767,the remainder
is the same in both the cases.What is the remainder?
a)101 b)458 c)757 d)cannot be determined
41) find the units place of the calculation 33342
23455
*789
678
+5
686

42) find the remainder of the calculation 42
123
/8.






10 seconds for aptitude



Calendars
This is a very interesting and puzzling topic for most students. The method discussed here is abstract
and very simple. Once you finish reading this section you will be able to find out day of the week for any
date from start of the calendar, be it Mahatma Gandhis day of birth or say 25
th
December of year 300028
within 30 seconds(average) and for the years that we regularly come across youll take 5 to 10 seconds.
Building the platform:
Before entering to the method and an example certain names and concepts should be made familiar.
These follow.
The Remainder Function:
The remainder of any division is written as the result, its called remainder function. (23/7)
r
=2
The subscript r indicates remainder function. (28/7)
r
=0, (31/4)
r
=3 . this should be clear before we proceed
further.
The Quotient function:
Only the whole number result is taken as result of the quotient function. (24/7)
q
=3 . The subscript q
indicates the quotient function. (18/4)
q
=4, (14/4)
q
=2, (94/4)
q
=23, (21/7)
q
=3.
10 seconds for aptitude
74

Concept of leap year:
Check if the last 2 digits of the year are divisible by 4, if they are it is a leap year. In case the last 2 digits are
00 then, check if the digits prior to them are divisible by 4, if they are it is a leap year (century year should
be a multiple of 400 to be a leap year).
1236 is a leap year (36 divisible by 4)
1300 is not (13 not divisible by 4)
1384 is a leap year (84 is divisible by 4)
1933 is not (33 not divisible by 4)
13200 is a leap year (132 is divisible by 4)





10 seconds for aptitude



Principle of counting:
Start counting from the bottom of your little finger, each joint corresponds to a number. The figure better
illustrates counting.


6


5 3

7

4
1

2




Principle of counting
Count from 1 to 7 as shown to count 8 come back to the position that holds 1 , 11 would be at the position
of 4.
Similarly the days of the week also correspond to these positions 1 mon, 2 tue, 3 wed. 7
sun. be very clear and practice at least 4 to 5 times before proceeding.
10 seconds for aptitude
76

KPs Rhyme:
This is the most important part without which the method doesnt work. Read it again and again till you
can remember it exactly in the same sequence.
my maths teacher taught me 3 + 3 is 6,
In my exam I wrote 1 + 4 as 6,
The exam was for 2 5 marks and I got 0,
That is why I couldnt score 3 5% in 1
st
attempt.
Observe the numbers underlined, there are 12 numbers corresponding to 12 months of the year.
-} A date 12/5/1950 means that from the start of the calendar 1949 years, 4 months are completely over
and it is the 12
th
day.
The method: it is explained with 3 examples stage by stage.
Ex1:- to find day of the week of 21/09/2011
2011 is the present year, 2010 years are completely over
9
th
month is going on, 8 are over
It is the 21
st
day.
10 seconds for aptitude



2010 8 21
(2010/400)
r
=
10

(10/4)
q
=2
2+10 = 12
(12/7)
r
=5
Start counting
on your fingers
as described in
the principle of
counting, it will
Stop at fifth bit
leave it there.
Do not remove
the thumb
placed on fifth
bit.
Remember the
KPs Rhyme,
the 8
th
number
in sequence is 5
5
Continue
counting with
the help of
thumb on your
fingers
6,7,8,9,10 it
will end on the
3
rd
bit keep
your thumb
there.
(21/7)
r
= 0
0
Add nothing to
your fingers.
The bit that
you lie your
thumb on is the
last bit bit of
little finger and
that
corresponds to
Wednesday
based on
principle of
counting.

So, 21/09/2011 is a Wednesday.
Ex2:- a small addition when the year to which the date belongs is a leap year.
15/08/2012
10 seconds for aptitude
78

2011 years are over
7 months are over
It is the 15
th
day.

2011 7 15
(2011/400)
r
=11
(11/4)
q
+11 = 13
(13/7)
r
=6
3
3
6
1
4
6
2
5
0
3
5
1
Nos are in order as
in the rhyme from
the second number
all the numbers are
one more than in
the rhyme for a leap
year.
7
th
no is 2 but since
it is a leap year, 3 is
taken.
3
(15/7)
r
=1
1
Count 6,3 and 1
on your fingers
it ends at the
last bit of little
finger, hence
Wednesday
10 seconds for aptitude



Initially we should always check if the year is a leap year or not. For leap year theres a slight change in the
months code everything else is the same.
Ex3:- 15/08/1947
100 200 300
5 3 1
1946 years are over
7 months are over
It is the 15
th
day.
1946 7 15
(1946/400)
r
=346
Additional step
when the
reminder
exceeds 2 digits,
346
300 + 46
From table above 300
=> 1
The 7
th
number
in the series is
2
(15/7)
r
=1
finally adding
this too you see
that thumb
indicates that it
is the 5
th

position,
Friday.
10 seconds for aptitude
80

Add 1 to your finger ,
continue as usual.
(46/4)
q
+46 = 57
(57/7)
r
= 1


Different questions on calendars along with simple methods to solve them
:-} If day before yesterday was Sunday 90 days from today will be?
Sol: since there are 7 days in a week, the same day will repeat after 7 days. Here, we use the remainder
function that gives output corresponding to number of days ahead from today. If day before yesterday was
Sunday, today is Tuesday. (90/7)
r
=6. Hence, the day will correspond to 6 days from today(Tuesday) that is
Monday.
-} Find the day of the week corresponding to following dates
a) 10/10/2011
b) 7/5/1947
c) 8/1/2044
d) 21/9/1994
10 seconds for aptitude



Sol: a) 10/10/2011
2010 years are over
9 months are over
It is the 10
th
day.
2010 9 10
(2010/400)
r
=10
10 + (10/4)
q
=12
(12/7)
r
= 5
9
th
number in
code is 0
(10/7)
r
=3
It ends on first
bit of little
finger Monday
b) 7/5/1947
=> 1946 years are over
=> 4 months are over
=> its the 7
th
day
1946 4 7
(1946/400)
r
=346
300 + 46
300 => 1
Remaining 46,
4
th
number is
1
(7/7)
r
=0
Last bit of
little finger
that is
10 seconds for aptitude
82

(46/4)
q
+46= 57
(57/7)
r
=1
Wednesday

c) 8/1/2044, 2044 is a leap year
=> 2043 years are over
=> 0 months are over
=> it is 8
th
day
2043 0 8
(2043/400)
r
=43
(43/4)
q
+43=53
(53/7)
r
=4
0 so, nothing
added.

(8/7)
r
=1 it
corresponds
to a Friday.

d) 21/9/1994, 1994 is not a leap year
=> 1993 years are over
=> 8 months are over
=> It is the 21
st
day
10 seconds for aptitude



1993 8 21
(1993/400)
r
=393
300+93
300 => 1
(93/4)
q
+93=116
(116/7)
r
=4
8
th
number in
series is 5
(21/7)
r
=0
Thumb
indicates last
bit of middle
finger,
Wednesday.


Profit and Loss
We purchase various commodities from shops in our daily life. But, do we ever try to find the
percentage profit owner makes. Why sales of a shop are better than the others? How is it that a
shopkeeper is so rich, earns more than a techie?
Ram once visits Shyams shop and buys jeans and T shirt. The price tagged on it is 3000/-. Shyam has
put up an attractive offer that 20% discount is given on tagged price. Ram calculates 20% of 3000 = 600/-,
deducts it from the tagged price and buys it at 2400/-. Shyam had bought this pair at 2000/-. He calculates
his percentage profit. His profit is 2400 2000 = 400/-, percentage profit indicates what part of the
10 seconds for aptitude
84

investment is the profit. For an investment of 2000/-, he gets a profit of 400/-. Profit percent = 400/2000
*100 = 20% .

Amount Formulae
MP 3000/-
D% 20%
D 600/- D%/100*MP
SP 2400/- MP-D
CP 2000/-
P 400/- SP-CP
P% 20% P/CP*100
The tagged price is called marked price.(MP)
Discount percentage is applicable to MP.
Discount value (D) is based on MP.
Selling price (SP) is the price for which customer buys the article.
Cost price (CP) is the price at which shopkeeper buys it.
Profit is SP CP.
Profit and loss percentage is always calculated based on investment made (CP).

10 seconds for aptitude




-} A man sells a pen at a profit of 25%. Had he bought it at 30% less and sold it for Rs 4 less, he would have
gained 20%. Find cost price of the pen?
Sol:
Case1: take x as the CP of the pen.
P = 25/100*x (25% of CP)
SP = CP + P
SP = x + 0.25x = 1.25x
Case2 : brings it for 30% less
CP = x 30/100*x = 0.7x
SP = 1.25x 4 (sells at 4 Rs less)
Given, P% = 20% =( SP-CP)/CP*100.
Substitute and solve to find CP.
CP = 16.32/-.
10 seconds for aptitude
86

-} The selling price of 20 articles is same as the cost price of 21. Find the gain%.
Sol: assume CP of each article is 1Re.
CP of 21 articles = 21/-
SP of 20 articles = 21/-
SP of 1 article =
21
/
20
/- P =
21
/
20
1 per article.
P% = (
21
/
20
1)/1*100
= 1/20*100 = 5%.
-} A man loses 12% by selling a book for 246 Rs. What should be his selling price to gain 15%?
Sol:
CP SP
X 0.88x = 246
X 1.15x = ?

? = 1.15x/0.88x*246 = 312.42/- (**10 seconds**)
Note: if a man has a gain or loss of x% and he wants a gain or loss of y% then,
10 seconds for aptitude



New SP = (1
+
/
-
y/100) * old SP/(1
+
/
-
x/100)
+ for gain, - for loss.
-} A man loses 8% by selling an article for Rs 46/-. If he wants to gain 8% what should his SP be?
Sol: new SP = (1.08)*(46/0.92) = 54/-.
-} A shopkeeper gains 20% on an article by selling it at Rs 132. Another shopkeeper who had bought it at
same cost loses 10%. What is the SP of another shopkeeper?
Sol: SP
2
= 0.9*(132/1.2) = 99/-.
-} A man sells 2 articles for Rs 300 each. On one he gains 10% and on the other he loses 10%. Find his net %
gain or loss.
Sol: when we have equal percentage gain and loss, multiply them to find the net loss. There is always a loss
regardless of the SP.
Loss% = 10/100 * 10/100 = 1/100 = 1% loss.
-} A shop keeper sells 2 pens at Rs 100 each one with a gain of 25% and other with a loss of 25 percent.
Find the total cost price.
Sol: net loss = 25/100 * 25/100 = 6.25% loss
10 seconds for aptitude
88

Total SP = 200/-
Total CP = SP/(1-0.0625) = 200/93.75 = 213.33/-.
-} A businessman sells 25% stock on 15% profit, 2/3
rd
of the remaining on 10% loss. At what percent should
he sell the remaining to get a 5% overall gain?
Sol: Assume that he has 100 grams stock that costs 100/- he should sell it at 105 for overall gain.
Quantity SP
available sold
100 25 28.75 (15% P)
75 50 45 (10% L)
25 25 ? = 31.25
0
Sum of SP = 105
? = 105 28.75 45 = 31.25/-
P% = (31.25-25)/25*100 = 25%.


10 seconds for aptitude



Simple interest:
The simple interest(SI) is the extra amount more than the principle a person has to pay when he has
borrowed a sum of money.
The interest he pays depends on the rate of interest(r), principal he has borrowed(p), and time after which
he returns(t).

Time should always be taken in years.
Rate in percentage per annum
A = SI+P = amount

Compound interest:
After a certain period, if the interest for the period till then is taken into account, interest is calculated and
added to the principal to obtain the new principal, it is called compounding.
(

)


10 seconds for aptitude
90

n = number of compoundings per year.
CI = A P.
Recurring deposits:
A sum of money is invested every month in the post office or in banks for a certain fixed time called
the maturity period. When this period is over, the investor gets the total principal he has paid along with
interest.
Consider the case where a person has deposited Rs 100 every month for year at the rate of 6% per
annum.
1
st
months deposit should be given interest for 12 months, 2
nd
months deposit should be given interest
for 11 months and so on till the last month. Let number of months be n. here, n=12.


10 seconds for aptitude




Sum of all these interests,

(

) (
)


(
)

) (NA)




Normalized average is that value for which interest for a period of 1 month is equal to the interest of the
recurring deposit.
10 seconds for aptitude
92

Partnership:
Simple partnership:
A group of people together invest in a partnership where the amount of investment remains the
same for the entire period. At the end of this period profit is divided amongst them in the ratio of their
investment.
Compound Partnership:
When the amount invested or the period of time varies in the group, the profits are divided based on
time value of money.
The sum of money is multiplied by period of investment for each partner. The end values are in a particular
ratio. In this ratio the money is divided.
Ex:- Rahul, Priya and David enter into a partnership. Rahul invests 3000Rs for 8 months and reduces it to
2000 till end of the year. Priya invests 2500 for the whole year. Dvid invests 2000in the beginning but at the
end of 3 mnths he increases it to 4000. Find the ratio in which profit should be divided.
Sol: time value of money ratio
Rahul : Priya : David

10 seconds for aptitude



32 : 30 : 36
16 : 15 : 18 is the ratio in which profits should be divided.
Problems to be solved:
Braun invested a certain sum of money at 8% p.a. simple interest for 'n' years. At the end of 'n' years,
Braun got back 4 times his original investment. What is the value of n?

Shawn invested one half of his savings in a bond that paid simple interest for 2 years and received $
550 as interest. He invested the remaining in a bond that paid compound interest, interest being
compounded annually, for the same 2 years at the same rate of interest and received $605 as interest.
What was the value of his total savings before investing in these two bonds?

Ann invested a certain sum of money in a bank that paid simple interest. The amount grew to $240 at
the end of 2 years. She waited for another 3 years and got a final amount of $300. What was the
principal amount that she invested at the beginning?

Peter invested a certain sum of money in a simple interest bond whose value grew to $300 at the end of
3 years and to $ 400 at the end of another 5 years. What was the rate of interest in which he invested
his sum?

10 seconds for aptitude
94

A father left a will of Rs.35 lakhs between his two daughters aged 8.5 and 16 such that they may get
equal amounts when each of them reach the age of 21 years. The original amount of Rs.35 lakhs has
been instructed to be invested at 10% p.a. simple interest. How much did the elder daughter get at the
time of the will?
What will Rs.1500 amount to in three years if it is invested in 20% p.a. compound interest, interest
being compounded annually?
If a sum of money grows to 144/121 times when invested for two years in a scheme where interest is
compounded annually, how long will the same sum of money take to treble if invested at the same rate
of interest in a scheme where interest is computed using simple interest method?
The population of a town was 3600 three years back. It is 4800 right now. What will be the population
three years down the line, if the rate of growth of population has been constant over the years and has
been compounding annually?
A man invests Rs.5000 for 3 years at 5% p.a. compound interest reckoned yearly. Income tax at the
rate of 20% on the interest earned is deducted at the end of each year. Find the amount at the end of the
third year.
The difference between the compound interest and the simple interest on a certain sum at 12% p.a. for
two years is Rs.90. What will be the value of the amount at the end of 3 years?
A stairway 10ft high is such that each step accounts for half a foot upward and one-foot forward. What
distance will an ant travel if it starts from ground level to reach the top of the stairway?
A sum of money invested for a certain number of years at 8% p.a. simple interest grows to Rs.180. The
same sum of money invested for the same number of years at 4% p.a. simple interest grows to Rs.120
only. For how many years was the sum invested?
10 seconds for aptitude



How long will it take for a sum of money to grow from Rs.1250 to Rs.10,000, if it is invested at 12.5%
p.a simple interest?
Rs. 5887 is divided between Shyam and Ram, such that Shyam's share at the end of 9 years is equal to
Ram's share at the end of 11 years, compounded annually at the rate of 5%. Find the share of Shyam
The question for the day is from the topic simple and compound interest. Shawn invested one half of
his savings in a bond that paid simple interest for 2 years and received Rs.550 as interest. He invested
the remaining in a bond that paid compound interest, interest being compounded annually, for the same
2 years at the same rate of interest and received Rs.605 as interest. What was the value of his total
savings before investing in these two bonds?
Rs.100 doubled in 5 years when compounded annually. How many more years will it take to get
another Rs.200 compound interest?
How long does it take a principal of $25,000 at a simple interest rate of 5% to become $30,000?
$45,000 is deposited into a savings account. After one year, 4 months and 20 days it totals $52,500.
Calculate the simple interest rate for this account.
Determine the simple interest rate applied to a principal over 20 years if the total interest paid equals
the borrowed principal.
How long does it take a principal payment to triple at a simple interst rate of 6%?
Find the total amount of simple interest that is paid over a perod of five years on a principal of $
30,000 at a simple interest rate of 6%.
Calculate the total worth of an investment after six months with a principal of $10,000 at a simple
interest rate of 3.5%.
A sum of money deposited at C.I. amounts to $2420 in 2 years and to $2662 in 3 years. Find the rate
percent
10 seconds for aptitude
96

Find the sum lend at C.I. at 5 p.c per annum will amount to $441 in 2 years
A property decreases in value every year at the rate of % of its value at the beginning of the year. Its
value at the end of 3 years was $21093.95. Find its value at the beginning of the first year.
Find the least number of complete years in which a sum of money put out at 25% compound interest
will be more than double of itself?
A property decreases in value every year at the rate of % of its value at the beginning of the year. Its
value at the end of 3 years was $21093.95. Find its value at the beginning of the first year.
The difference between simple interest and C.I. at the same rate for $5000 for 2 years is $72. The rate
of interest is?








10 seconds for aptitude



Permutation and combinations
Principle of counting: considering a very simple example, we understand the principle of counting.
Ex: there are 4 cities A, B, C and D connected by many roads as shown in figure.

A B C D

A person has to go to D from A.
He has 3 ways to reach B first from A. then, any one way taken, he has 5 more ways to reach C from B.
from A to C he has 3*5 = 15 ways. Each of these 15 ways taken she has 2 more ways to reach D from C.
this gives rise to new possibilities = 15*2 = 30 ways to reach D from A. a person can travel from A to D in
30 ways(3*5*2).
If the whole event of reaching D from A has to be done then number of ways to reach D is given by
n1*n2*n3 where, n1 is the number of ways in which he can reach B from A. Number of ways from B to C is
n2 and for C to D is n3.
10 seconds for aptitude
98

Conclusion: if an event can happen in n1 ways and each of these can happen in n2 ways and further each
one can happen in n3 ways and so on. The number of ways in which the whole event can happen is
n1*n2*n3**nX ways.
Permutation (possible mutations):
Possible arrangements
To explain permutations, I consider this example:- we have 3 students, Amit(A), Prashant(P) and
Shashank(S). there are 2 chairs X and Y.
X Y
The number of ways in which they can be arranged in these chairs is called permutations.
Ways X Y
1 A B
2 A S
P P A
4 P S
5 S A
6 S P
Totally we have 6 ways in which 2 people can be seated. If these three people have to be arranged in only
one chair, then we have 3 arrangements.
10 seconds for aptitude



Now we observe from the example above that one person is fixed to one position first next the possibilities
for arrangements further are done. The second person is fixed in first place again and all possible changes
are done. When all three seats are empty A, P or S can be put in the first place. So we have 3 people and 3
ways in which any one of them can be fixed in the first position. Two places are left out and second
position can be filled by any one of the remaining 2 people. (if we fix A in the first place, P or S can be put
in the second place). When 2 people are seated, theres one person and one place left out that can be
selected in one way. Hence from the principle of counting, we arrive at the conclusion that 3 people can be
arranged in 3 chairs in 3*2*1 = 6ways.
Here, we denote such arrangements by the factorial expression. i.e, 3!
4!=4*3*2*1 and so on.
n! = n*(n-1)*(n-2)*(n-3)**1 .
Further analysis: till now we had 3 people hence, 3 ways to put a person on the first chair. If we have 4
people A, P, S and T we have four ways to select a person to sit on the first chair. Consider the situation
when 4 people have to sit in 2 chairs.
Ways X Y
1 A P
S A S
3 A T
10 seconds for aptitude
100

4 P A
5 P S
6 P T
7 S A
8 S P
9 S T
10 T A
11 T P
12 T S
Here first place can be selected in 4 ways and next place in 3 ways. Total no of possibilities is 4*3 = 12
ways.
For 4 places it is 4*3*2*1 ways, for 3 places it is 4*3*2 ways and for 2 places it is 4*3 ways.
4 people 4 places 4*3*2*1 ways
4 people 3 places 4*3*2 ways
4 people 2 places 4*3 ways
4 people 1 place 4 ways
Number of places is number of nos to be multiplied so to remove the other part from 4!.
It can be written as 4!/(4-4)!, 4!/(4-3)!, 4!/(4-2)!, 4!/(4-1)! .
10 seconds for aptitude



Conclusion:
If we have n people and r places, number of ways in which they can be arranged is given by n!/(n-r)!. this
expression is given a notation nP
r
and nP
rle
= n!/(n-r)!.
nP
r
is the number of permutations of n different things taken r at a time.
I have 2 similar books, a pen and an eraser. I can arrange them such that 1
st
place can be selected in 4
ways, second place in 3 ways and 3
rd
and 4
th
place are same since both are similar. Either of the book put in
this place gives the same result. So, number of possible mutations are 4!/2! Where, n objects out of which
x are similar. In general if out of n objects x are alike, y are alike and z are alike, then,
n!
/
x!y!z!
is the number
of permutations.
The ghost theory:
We have 3 small safe chairs and 5 people A,B,C,D and E. 3 among the 5 can sit on the chairs. There is
no go but to let the left out ones be eaten by the ghost. Now, if I have to choose a person to sit on the first
chair, since there are 5 people, I have 5 options. Now one person is seated. To choose the next person to
sit for the second chair, there are 4 people left so, 4 options. For the last chair similarly I have 3 options.
The total number of ways in which I can choose 3 from 5 is 5*4*3. This includes different arrangements in
same group of people also. i.e if I interchange person sitting in the first chair and the one sitting in the
second chair, it is considered as a new possibility. But, whatever be the arrangement inside the selected 3
10 seconds for aptitude
102

they are safe from the ghost. To find no of different groups that I can make I have to divide all possible
ways by number of arrangements in a single group. The number of possible ways of arranging 3 people in
group into 3 chairs is 3*2*1.
Number of groups =
number of ways
/
(number of ways per group)
Number of groups = 5*4*3 / 3*2*1 =10 groups.
Out of these 10 groups one of them is selected to be saved from the ghost. This is called combinations. 5C
3

is its representation.
5*4*3 = 5!/(5-3)! And 3*2*1 = 3!
Hence, 5C
3
= 5!/(5-3)!3! in general,
nC
r
= n! / (n-r)!r!
nC
r
= nP
r
/r!
-} A committee is to be formed from 7 engineers and 4 technicians. How many different commities can be
made if there are 3 engineers and 1 technician in a committee.
a) with no restriction b) 2 engineers in particular should be selected c) 1 technician in particular to be
selected.
10 seconds for aptitude



Sol:
a) number of combinations of engineers is given by 7C
3
= 35.
Number of combinations of technicians=3C
1
=3
Total number of ways = 7C
3
*3C
1
= 35*3 = 105.
b) If two engineers must be selected, disregard them and select from the remaining. 1 engineer is to be
selected from the remaining 5. So, 5C
1
=5ways. I technician is to selected number of ways are 3C
1
=3.
Total number of ways = 5*3 = 15 ways.
c) if one technician has already been selected, then nothing else can be done in the technicians part. So 3
out of 7 engineers have to be selected. Hence 7C
3
=35 ways.

NUMBER OF PERMUTATIONS UNDER CERTAIN CONDITIONS
1. Number of permutations of n different things, taken r at a time, when a particular thng is to be always
included in each arrangement , is .
2. Number of permutations of n different things, taken r at a time, when a particular thing is never taken in
each arrangement is .
10 seconds for aptitude
104

3. Number of permutations of n different things, taken all at a time, when m specified things always come
together is .
4. Number of permutations of n different things, taken all at a time, when m specified never come together
is .
5. The number of permutations of n dissimilar things taken r at a time when k(< r) particular things always
occur is .
6. The number of permutations of n dissimilar things taken r at a time when k particular things never occur
is .
7. The number of permutations of n dissimilar things taken r at a time when repetition of things is allowed
any number of times is
8. The number of permutations of n different things, taken not more than r at a time, when each thing may
occur any number of times is .
CIRCULAR PERMUTATIONS
}1. The number of circular permutations of n dissimilar things taken r at a time is .
2. The number of circular permutations of n dissimilar things taken all at a time is .
3. The number of circular permutations of n things taken r at a time in one direction is .
10 seconds for aptitude



4. The number of circular permutations of n dissimilar things in clock-wise direction = Number of
permutations in anticlock-wise direction = .

Probability defining chance

Any date selected at random will correspond to a day of the week, that is from Monday to Sunday.
There are seven days in a week. So if a date is selected at random you cannot be sure that it is a Sunday.
Now, how sure can you be? If I have a coin and toss it, then I cannot be 100% sure that it will lie heads on
top. So, to what percent can I be sure? The measure of this percent is defined as probability.
The coin has a head and a tail, if a head turns up at the same time tail cannot turn up. So, these two
are called mutually exclusive events. I cannot expect a third possibility other than head and tail, so they are
exhaustive. I also cannot say that it would always turn heads up rather than tails, so both head and tail are
equally likely events.
Similarly when I toss a die with six faces marked with 6 different numbers. 1 out of 6 turns up. This is
an equally likely mutually exclusive and exhaustive event. Since I have only 6 possibilities, no two can occur
at the same time and none of them can occur in preference to the other. In tossing a coin, if
10 seconds for aptitude
106

I want head to turn up that is one possibility out of 2 options possible(heads and tails). I can be half sure
that it would be heads only. So possibility of head is 50% or out of 1. From now on we consider 100% as
1 and our chance % as a fraction of 1. Since I had 1 event (heads turning up) in my favor, I call it odds in
favor of the event.
Now, me winning is the chance.
P(head) =
1 favorable
/
2possible
. =
In a die if I want 6 to turn up,
P(6) =

1 favorable
/
6 possible
. = 1/6 .
If I do not want 6 to turn up,
I can remove probable occurrence of 6 from the total. P(not getting 6) = 1(total) -
1 unfavorable
/
6 possible
. -
= 5/6.
5/6 is the probability of a face other than 6 turning up. It can also be considered as 5 favourable cases by 6
possibilities. We can observe that sum of probability of getting 6 and not getting 6 is 1 that is total
probability. 5/6 + 1/6 = 1.
P(A) + P(A) = 1.
A = against my wish, A favourable.
10 seconds for aptitude



If I need any one of the 6 faces to turn up, then, all 6 possibilities are in my favor. Hence, P=6/6 = 1.
In case I do not want any of the face to turn up, it cannot happen hence, 0 favorable out of 6 possibilities. P
= 0/6 = 0.
Probability that an event A occurs after event B has occurred is called conditional probability P(
A
/
B
).

Problems
In how many ways can the letters of the word ABACUS be rearranged such that the vowels always
appear together?
How many different four letter words can be formed (the words need not be meaningful) using the letters
of the word MEDITERRANEAN such that the first letter is E and the last letter is R?
What is the probability that the position in which the consonants appear remain unchanged when the
letters of the word "Math" are re-arranged?
There are 6 boxes numbered 1, 2, ... 6. Each box is to be filled up either with a red or a green ball in such
a way that at least 1 box contains a green ball and the boxes containing green balls are consecutively
numbered. The total number of ways in which this can be done is:

A man can hit a target once in 4 shots. If he fires 4 shots in succession, what is the probability that he will
hit his target?
In how many ways can 5 letters be posted in 3 post boxes, if any number of letters can be posted in all of
10 seconds for aptitude
108

the three post boxes?
Ten coins are tossed simultaneously. In how many of the outcomes will the third coin turn up a head?
In how many ways can the letters of the word "PROBLEM" be rearranged to make seven letter words
such that none of the letters repeat?


Speed, time and distance
The three words in the heading are interrelated. Speed is the distance that an object travels in a second. In
mathematical terms,
Speed = distance travelled/time taken
To analyze different situations we consider a factor to be constant and observe variations in the other two.
In a race, distance to be travelled is same for all participants. Hence, the variables are speed and
time.
We know that a person who has higher speed will take lesser time. (product of speed and time is constant,
when one of them increases the other automatically decreases)
10 seconds for aptitude



d = s*t = s1*t1 = s2*t2, where d is the distance, s,s1,s2,t,t1,t2 are variables with respect to different
participants.
S1/s2 = t2/t1. Speed and time are inversely proportional.e taken by them is in ra
-} In a race for 2 participants, the slower person finishes 10 seconds after the winner. If speed of winner is
10 m/s find speed of runner. Given time taken by them is in ratio 6:7. Find length of the race.
Solution: ratio of speeds of the 2 people, s2/s1=s2/10 = t/t+10 = 6/7 (time taken by winner be t seconds)
From the above equation, s2=60/7, this is speed of 2
nd
person
7t = 6t+60 => t=60 seconds. First person has travelled for 60 seconds at 10m/s hence length of the
race = 10*60 = 600m.
When the time given is constant the distance covered is larger with larger speed.
D1/D2 =S1/S2. In a particular time given, ratio of distance covered is equal to the ratio of their speeds.
-} Two cities A and B are 100km apart, Buses X and Y leave from them respectively towards each other.
They meet after 5 hours at a point. If their speeds are in ratio 3:1, find the distance from B at which they
meet? Find their speeds?
Solution : speeds are in ratio 3:1 and time for which they travel till they meet is constant so, distance
travelled by them is in the same ratio.
10 seconds for aptitude
110

Dividing the distance 100km in ratio 3:1 we find that x has travelled 75km and y has travelled 25km.
Speed of X = 75km/5hr = 15km/hr.
Speed of y = 25km/5hr = 5km/hr.
Speed when considered constant, the ratio of time taken is same as is of distance travelled.
d1/d2=t1/t2
-} A person walks from his home to college for 1 hour. If he travels for another 2 hours, he can reach a park
in the same direction. If the park is 20km from his college, what is the distance between his house and the
park?
Solution:
distance
park to house
/distance
park to college

= time
park to hose
/time
park to college

?/20 = 2+1/2.
? = 30km.


10 seconds for aptitude



Average speed:
Ram drives a bike for 25km on a ordinary road. He has slowed down many times and the speed was
between 10 and 50km/hr. finally if he should find in what time he can cover 15km on a similar road he
needs the concept of average speed.
Average speed = total distance/total time
In case the journey is for different durations at different speeds , find the individual distances add
them and divide by sum of time to get the average speed.
s1*t1 + s2*t2 + s3*t3 +/t1+t2+t3+
If equal distances are travelled at different speeds as in the case of a person moving up and down the hill.
Then, average speed is given by s
avg
=2s1*s2/(s1+s2). Here s1 and s2 are speeds at which he travels uphill
and then downhill respectively.
-} Rahul goes to his native 120km far in 6 hours and comes back with an average speed of 30km/hr. what is
the average speed for the entire journey?
Solution: while going to his native,
Speed = 120km/6hr = 20km/hr.
Speed
return
= 30km/hr
10 seconds for aptitude
112

S
avg
= 2*20*30/(20+30) = 24km/hr.
Average speed fot the entire journey is 24km/hr.
-} The distance between two cities A and B , between B and C is in the ratio 7:9. A person travels at
56km/hr and 45km/hr respectively. What is his average speed from A to C
Solution: to solve this problem, assume the distance to be 7 and 9km itself. (7 and 9 from ratio) now
compute time taken.
t
AB
= 7/56 = 1/8 hrs and t
BC
= 9/45 = 1/5 hrs
s
avg
= total distance/total time
= (7+9)/(1/8+1/5)
= 640/13 = 49
3
/
13
km/hr.
**10 seconds**
If r1:r2 is ratio of distances travelled,
S
avg
=(r1+r2)/(r1/s1+r2/s2)

10 seconds for aptitude



Relative speed
When two objects move with different speeds, the distance of separation between them changes
continuously. The change that occurs per unit time is called relative speed. If they are moving together in
the same direction at same speed, distance of separation does not change relative speed is zero. If two
objects move such that they have constant distance of separation, even then relative speed is zero.
Relative speed = change in distance/time
-} two buses move from the same city A towards B and C. B is to the west of A and C to the east. If they
reach at 6p.m. and 8p.m. respectively starting at 4p.m, given bus to B is at 60km/hr and their speeds are in
ratio 20:21, find the distance between B and C.
Speeds are in ratio 20:21
20/21 = 60/s2
S2 = 63km/hr
D1
(B)
= 2hrs*60km/hr = 120km.
D2 = 4hrs*63km/hr = 252km
Total distance = 120+252 = 372km.
-} Two buses move at 43km/hr and 63km/hr towards each other, if they meet after 3 hours what was their
initial distance of separation?
10 seconds for aptitude
114

Solution: Relative speed = 43+63 = 106km/hr
Distance=speed*time = 106*3 = 318km.
-} A leaves from B towards C at 6a.m. one hour later D leaves B at 10% higher speed. They meet at a
distance of 25km at 9a.m find their speeds?
Solution: if A travels at x km/hr, D travels at 1.1x km/hr. relative speed = .1x km/hr
Relative speed = change in distance/time
= (25km 0km)/2hrs
= 12.5km/hr
0.1x = 12.5 km/hr
x = 125 km/hr , 1.1x = 137.5 km/hr

Objects crossing each other in a particular time:
Rakesh observed that a train crossed a pole in 10 seconds. How can he find speed of the train? The
explanation further helps us find it.
Master formula:

s
1
+/- s
2
= (l
1
+l
2
)/t
10 seconds for aptitude




s1 = speed of first object
s2 = speed of the other object
l1 = length of first object
l2 = length of second object
t = time for which they cross each other.
+ = opposite direction, - = same direction.
Problems on trains:





10 seconds for aptitude
116

Case1: consider a train crosses a pole in time t seconds, where length of the train is L
t
and speed of train is
S
t
.
Time starts:

Time ends:


The point F on the train is displaced by a distance equal to length of the train in t seconds.
Hence, S
t
= L
t
/t.

Case2: a train crosses a railway platform
The first point F on the train is displaced by a certain distance while the clock ticks and train crosses
the platform. This distance is equal to length of the train + length of the platform.
Start:
10 seconds for aptitude






End:
L
p
+ L
t



Speed of train = (L
p
+ L
t
)/t
Using the master formula,
s
1
+/- s
2
= (l
1
+l
2
)/t , s1 is speed of train , s2 = speed of platform = 0, l1 = length of train, l2 = length of
platform and t is time in seconds.
** This formula is universal and can be used in many situations. Practicing with it is recommended to
improve pace**
Case3: two trains cross each other
10 seconds for aptitude
118

start:


f
end

f

here, distance travelled = sum of lengths of the trains, relative speed is sum of speeds.
S
t1
+ S
t2
= (L
t1
+L
t2
)/t
Task : analyze the case in which a train crosses a moving person and a bus overtakes another bus. Then
youll appreciate the use of master formula.
-} A train crosses a signal in 10 seconds while moving at 25m/s and subsequently crosses another train
moving in opposite direction in 12 seconds. If length of other train is 400m find its speed?
Sol: case1, train crosses the signal
S
t
= L
t
/t => 25 = L
t
/10
10 seconds for aptitude



L
t
= 250m.
Case2, two trains cross each other
S
t1
+ S
t2
= (L
t1
+L
t2
)/t
25 + S
t2
= (250+400)/12 = 650/12 => S
t
= 29
1
/
6
m/s.
-} A train crosses a platform in 12 seconds and subsequently crosses a railway bridge in 20 seconds. If
length of the railway bridge is 1280m, find length of platform, given that train is 120m long.
Sol: using ratio
Since time is given in both the cases, speed of train is constant , ratio of distance is same as ratio of time.
Train crosses bridge, distance covered = 120+1280 , train crosses platform, distance covered = 120+L
p
.
Ratio of time taken is 20:12
(120+L
p
)/(120+1280) = 12/20
L
p
= 720m.


10 seconds for aptitude
120

Task: solve this question.
A railway line inspector leaves from city A along with a train leaving to city B at 6:00 a.m to check if there is
a train coming from city B on the same track. He travels a distance of 120km and finds another train
coming towards A. he shows a red flag and comes back towards A and show red flag to the other train also.
Both the trains stop such when distance between them is 10km at 12 noon. If a train comes to halt 10 km
after signal is shown and speeds of both trains are same. Find distance between A and B.
Sol: **10 seconds**
120km
T1 T1 final T2 final T2
A B ` 5km 10km 5km X km






10 seconds for aptitude



Circular tracks:
When two objects move at different speeds starting from the same point in a circular track, they meet
each other after some time. Ex:- winner of a 800m race was very fast and one lap ahead of the guy who
finished last. The faster person completes one circle more than the slower one and will be together with
him again. Analyzing this situation, when the race started the distance of separation between them was
zero. Finally the faster one is one circle ahead i.e , he has travelled extra length equal to length of track. If
R
s
is the relative speed, then time t after which they meet is given by, t = length of circle/R
s
.
If 3 people start together, time after which the first two meet is found, time when 1
st
and 3
rd
person
meet is found. Then LCM of these two values of time gives us the time after which all three of them are
together for the first time.
-} A, B and C start from the same point with speeds 10m/s, 12m/s and 15m/s respectively. If they are
moving in the same direction on a circular track 2400m long, after what time will all of them meet.




10 seconds for aptitude
122

Sol:
3 sec later





1 sec
Start
Distance of separation between them increases with time due to difference in speeds.
Time after which A and B meet,
T1 = length of track/relative speed = 2400/2 = 1200 seconds.
Time after which B and C meet,
T2 = 2400/(15-12) = 800seconds.
10 seconds for aptitude



This means that A and B meet for every 1200 seconds, B and C meet for every 800 seconds. A and B
meet for the second time at 2400 seconds. B and C meet for third time after 2400 seconds hence all of
them are together again. The answer 2400 is found easily by taking LCM of 800 and 1200.
-} A and B travel in opposite directions at 24m/s and 16m/s on a 200m circular track. What is the distance
travelled by B when they meet for the second time?
Sol: since they travel in opposite directions, relative speed = 24+16 = 40m/s.
Time after which they meet,
T = 200/40 = 5 seconds.
When they meet for the second time, time elapsed is 5*2=10 seconds. In 10 seconds B has travelled a
distance equal to speed of B*10. = 16*10 = 160m.





10 seconds for aptitude
124

Boats and streams:
A stream carries anything that is freely floating on it in the downstream direction at speed equal to
speed of the stream.
The speed of a boat while moving in still water(pond, lake) is the actual speed of the boat. When it
moves in a stream, depending upon direction of movement of the boat , the stream makes it move further
more or move lesser.
Consider a boat moving downstream where speed of stream is 2m/s and speed of boat in Stillwater is
5m/s. if this boat starts to move from a point on the stream and at the same time a leaf is dropped into the
stream, then after one second the leaf is 2m ahead of its initial position due to speed of the stream and the
boat is 5m from the leaf i.e, 7m from its starting point. The distance travelled by boat in one second is its
speed = 7m/s in the stream.
Speed of boat downstream = speed of boat in still water + speed of the stream.




10 seconds for aptitude



Escalators:
An escalator has steps that continuously move in a direction. If an escalator takes 10 steps in a
minute, a person standing on it is raised by 10 steps in a minute. If there are 30 steps on the escalator then
in 3 minutes he will reach the top.
In case the person also climbs up the escalator ?? To understand this situation let us first consider a
simple staircase where S1 is the first step, S2 is the second and so on. Say, the person takes 5 steps in a
minute. He would be on S5 after one minute, when he comes to S30 he has reached the top.
If he climbs on an escalator, one minute later, the step S1 would be 10 steps higher after a minute
and the person would be on S5. Totally 10+5 = 15 steps are covered in a minute. To climb 30 steps, he will
take 2 min.
Consider the person and escalator is moving in opposite directions. If the person starts from S10, he
would be on S5 after 1 minute and the step S10 would be 10 steps higher. Net effect is that the person still
moves in the direction of the escalator at 5 steps per minute.
:-} An escalator has 32 steps which moves upwards at 8 steps per minute. A person walks at 12 steps per
minute. How many steps should he take to reach the bottom of the escalator from the top?
Sol: net no of steps covered per minute
10 seconds for aptitude
126

= 12-8 = 4 steps.
Time taken = 32steps/4 in a min = 8 min
Steps the person takes in 8 min = 8*12
= 96 steps.
-} A person takes 14 steps by the time escalator takes 6 steps. If the esacalator has 40 steps, how many
steps should he take to climb up and get down?
Sol: in same direction steps covered
= 14+6 = 20steps per minute
Time to reach the end = 40/20 = 2 minutes
Steps covered in the opposite direction
= 14-6 = 8steps per minute
Time to reach the end = 40/8 = 5 minutes
Total time taken = 2+5 = 7 minutes.
Number of steps taken in this time = 14*7 = 98 steps.
10 seconds for aptitude




A set of typically asked questions from the topic "Speed Time Distance" that acquaints you to different
concepts in the topic including relative speeds, average speeds, different units for measurement of speed,
time and distance and the conversions of these units. Also includes questions on boats in streams. Its an
important topic. Aspirants definitely have to master it.
A train traveling at 72 kmph crosses a platform in 30 seconds and a man standing on the platform in
18 seconds. What is the length of the platform in meters?
A train traveling at 100 kmph overtakes a motorbike traveling at 64 kmph in 40 seconds. What is the
length of the train in meters?
Jim travels the first 3 hours of his journey at 60 mph speed and the remaining 5 hours at 24 mph
speed What is the average speed of Jim's travel in mph?
A man can row 50 km upstream and 72 km downstream in 9 hours. He can also row 70 km upstream
and 90 km downstream in 12 hours. Find the rate of current.
Ram covers a part of the journey at 20 kmph and the balance at 70 kmph taking total of 8 hours to
cover the distance of 400 km. How many hours has been driving at 20 kmph?
Yana and Gupta leave points x and y towards y and x respectively simultaneously and travel in the
same route. After meeting each other on the way, Yana takes 4 hours to reach her destination, while
Gupta takes 9 hours to reach his destination. If the speed of Yana is 48 km/hr, what is the speed of
Gupta?
By walking at 3/4
th
of his usual speed, a man reaches office 20 minutes later than usual. What is his
usual time?
10 seconds for aptitude
128

I travel the first part of my journey at 40 kmph and the second part at 60 kmph and cover the total
distance of 240 km to my destination in 5 hours. How long did the first part of my journey last?
A passenger train covers the distance between stations X and Y, 50 minutes faster than a goods train.
Find this distance if the average speed of the passenger train is 60 kmph and that of goods train is 20
kmph.
The speed of a motor boat itself is 20 km/h and the rate of flow of the river is 4 km/h. Moving with
the stream the boat went 120 km. What distance will the boat cover during the same time going
against the stream?
A man and a woman 81 miles apart from each other, start travelling towrds each other at the same
time. If the man covers 5 miles per hour to the women's 4 miles per hour, how far will the woman
have travelled when they meet?
Two boys begin together to write out a booklet containing 535 lines. The first boy starts with the first
line, writing at the rate of 100 lines an hour; and the second starts with the last line then writes line
534 and so on, backward proceeding at the rate of 50 lines an hour. At what line will they meet?
A train traveling at 72 kmph crosses a platform in 30 seconds and a man standing on the platform in
18 seconds. What is the length of the platform in meters?
A train traveling at 100 kmph overtakes a motorbike traveling at 64 kmph in 40 seconds. What is the
length of the train in meters?
Jim travels the first 3 hours of his journey at 60 mph speed and the remaining 5 hours at 24 mph
speed. What is the average speed of Jim's travel in mph?
10 seconds for aptitude



A runs 25% faster than B and is able to give him a start of 7 meters to end a race in dead heat. What
is the length of the race?

Jane covered a distance of 340 miles between city A and city taking a total of 5 hours. If part of the
distance was covered at 60 miles per hour speed and the balance at 80 miles per hour speed, how
many hours did she travel at 60 miles per hour?

Steve traveled the first 2 hours of his journey at 40 mph and the last 3 hours of his journey at 80 mph.
What is his average speed of travel for the entire journey?








10 seconds for aptitude
130

Theory of averages and allegations

As we all know average can be defined as the sum of all the elements divided by the numbers of elements
present. Thus,



So, if I have some elements as 63, 57, 79, 83, 68 what will be the average?


Simple enough, RIGHT??? Absolutely!!!!
Now, let me say, the above mentioned is just the way to find the average, Its not the definition.
Average can be defined as the representation of a group, such that if all the elements are represented and
replaced by the average, the sum/totality of the group will not change. One must understand the
definition of the averages to apply it conveniently.
Lets observe,
63 57 79 83 68 Sum = 350
70 70 70 70 70 Sum = 705
Lets try to reason it out. Observe carefully.
Elements 63 57 79 83 68 Sum = 350
Average 70 70 70 70 70 Sum= 705
Deviations -7 -13 +9 +13 -2 Sum= 0
Here the last row gives the deviation of the elements from the average, as 63 is 7 less than 70, so -7, 57 is
13 less than 70 and so on. Now here we can observe that the various deviations of the elements gets
10 seconds for aptitude



canceled among themselves (-7 -13 +9 +13 -2 = 0), and hence the overall deviation is zero. And hence the
sum of the quantities remains same.

So, if we have n elements as e
1
, e
2
, e
3
, , e
n-1
, e
n
; and there average is a, then


Now, how is this definition helpful?
This definition gives us the idea that average is a pivotal quantity about which the elements balance
themselves. This will also lead to the conclusion that average should lie in between the greatest and the
lowest value i.e. the average cant be more than the greatest value ( 83 in the above example) and it cant
be less than the lowest value (57 in the above example). Lets use this definition to find the averages.
Solved Example: Find the average of 145, 179, 163, 182, 169, 173.
By just observing the numbers, we can understand that average should lie in between 145 and 182,
somewhere close to 170. Lets assume the average as 169 (one of the element itself) , and lets try to find
the deviation of the sum from the assumed sum.
145 179 163 182 169 173
169 169 169 169 169 169
-24 10 -6 13 0 4 Sum= -3
So total deviation is -3, if equally distributed among all those six 169s, each will get -0.5 and hence the
actual average will become 168.5.
10 seconds for aptitude
132

Isnt canceling the deviations easier then adding up the big numbers up there? And as you will practice this
method more, you will realize that the more accurate is your approximation the lesser calculation will be
involved in.
Solved Example: Average percentage scored by Nandini in her seven semesters is 78. How much she must
score in her eighth semester so as to have her overall average as 80?
As per the definition discussed above, we can assume that she would have scored 78 in all her seven
semesters. Now if she wants to have 80 as the overall average ( that means 80 in all eight semesters), she
should compensate for those two marks, apart from scoring 80 in her eighth semester.
So she must score 80 + 2 7 = 94.
Solved Example: Average percentage scored by Nandini in her seven semesters is 78. How much she must
score in her eighth semester so as to have her overall average as 75?
75 3 7 = 51
Properties of the Averages:
1. The average should lie between the largest and the smallest element of the group.
2. Middle term OR Average of the middle terms is the average of an Arithmetic Progression.
E.g. the average of 121, 129, 127, 125, 123, 131 is 126 i.e. average of 125 and 127 (though the
elements are not in the order but they are still the elements of an AP).
3. If every elements of a group is increased (or decreased) by the a constant, then the average also gets
increased (or decreased) by the same constant. E.g. if the average age of a family presently is 35
years, then the average age of the family 5 years hence will be 40 years irrespective of the number of
members of the family (Of course subject to condition that no one joined or left the family).
10 seconds for aptitude



4. If every element of a group is multiplied (or divided) by a constant, then the average will also get
multiplied by the same constant.
Solved Example: 6 years ago the average age of a family of six members was 47 years. 3 years ago one
person of the family got married and hence a 22 years old beautiful bride joined the family. This year the
grandpa aged 82 passed away. Find the new average age of the family?
The average age 3 years ago, before the marriage, should be 50 (using the concept discussed above).
The age of the new bride is 28 less than the average, equally distributed among the seven members (bride
included) of the family, the new average after the marriage will be 50 4 = 46.
Three years later i.e. this year, before the grandpa passed away, their average age will be 49.
The age of grandpa is 33 years more than the average, equally distributed among the six member
(excluding the grandpa) of the family, the new average will be 49 - 5.5 = 43.5
Weighted Averages:
Now, let me have 2 students in section A, and 3 in section B of a class. Let the average marks scored by
each of the students of the section A is 60 and that of the section B is 80. Then what is the over all average
of the class?
Conveniently applying the above discussed definition of averages, we can represent the score of students
of section A as 60, 60 and that of section B as 80, 80, 80. Total number of elements is 5. Now taking the
average of these marks:

Now, if the number of students in section A and B is 20 and 30 respectively, then what will be the overall
average of the class?
10 seconds for aptitude
134

Almost all of you must have guessed it correct. 60 will repeat 20 times and 80 will repeat 30 times, and the
total number of elements in this group would have been 50.

So, what if section A has 40 students and B has 60?

Some of you might have observed that the average in all the three cases is the same. Simple reason being
the ratio of the number of students in each of these sections in 2:3. These numbers, 2 and 3, here
represents the weights or strengths of the section A and B, and if one uses the weights instead of actual
number of elements in the group, the concept is called as Weighted Average. So instead of considering
the values as 40 and 60, one can conveniently take the values as 2 and 3, and get the same results.
We can compare this scenario with a tug of war game where both the quantities (60 and 80 as in our
previous example) intend to pull the average towards its side. And the position of the average will be
decided by strengths of these elements. If the strengths would have been equal i.e. the average would
have been exactly at the center (e.g. if the number of students in each of these sections would have been
23 and 23, the ratio of the weights would have been in the ratio 1:1, and the average would have been 70,
exactly at the center). Now, obviously if weight or strength of one element is more than that of the other,
it will pull average towards its side. In our example, as the strength of 80 is 3 as compared to 2 of 60, hence
the average is 72, nearer to 80 than that of 60. So, strength is inversely proportional to the difference (or
distance) of the element from the average. Greater the strength, lesser will be the distance.

Solved Examples
10 seconds for aptitude



Question
The average age of 10students in a class is increased by 2 year when two students aged 12 year and 14
year are replaced by 2girls . Find the average age of the two girls.
Solution
Let the sum of the ages of the 2girls be y years, also sum of the ages of the 2 students who have replaced =
26 years average age of group has gone up by 2 years that means the increase in total age of the 10
students is 20 years this increase is due to the age of the 2 girls hence y 26 = 20 , y= 4b therefore average
age of the 2 girls
Question
There are 60 students in a class . There students are divided into there groups A,B and c of 15, 20 an 25
students each . These groups A and C are combined to form group D. What is the average weight of the
students in group D?
(1). More than the average weight of A
(2). More than the average weight of C
(3)Less than the average weight of C
(4) Cannot be determined.
10 seconds for aptitude
136

Solution
Number of students in group D is more then number of students in group A or group C. But there is no
information about the weight of students in group A or group C. Hence answer is (4)
Note : As a group D has students form group C whichever group has higher average weight , the average
weight of group D will be that group s average weight
Question
If one student from group A is shifted group B, which of the following will be true?
(1) The average weight of all the four groups is same .
(2) The average weight of both the groups decrease.
(3) The average weight of the clear remains same
(4) Cannot to be determined.
Solution
Shifting students form groups A to group b , the total number of students in the clean remains same ,
hence
(3)is the correct answer.
10 seconds for aptitude



Question
If all the students of the class have the same weight , then which of the following is not true?
(1) The average weight of all the four groups is same.
(2) The total weight of A and C is twice the total weight of B.
(3) The average weight of D is greater then the average weight of A.
(4)The average weight of all the groups remains the same even if a number of students are shifted from
one group to another
Solution
Weight of each students is same is same therefore average weight of clean is same as average weight of
group A,B,C and D.
Hence (3) cannot be true.
Hence option (3) is correct answer.


10 seconds for aptitude
138

Allegations(mixture of mixtures)
We have 2 containers that have milk with water mixed in different ratios. In the 1
st
container
Water : milk :: 1:7 and in the 2
nd
container,
Water : milk :: 1:10. In what ratio should the contents from the containers be mixed to have resulting ratio
1:8?
Sol:-
Convert all the fractions into same base first
To convert take the LCM of denominators express ratios as a fraction of the denominator.
1
st
container water/total = 1/(1+7) = 1/8
th

2
nd
container water/total = 1/(1+10) = 1/11
th

Resulting mixture, water/total = 1/(1+9) = 1/10
th

=> Take LCM of 8, 11 and 10. = 440
=>


10 seconds for aptitude



Write them as shown below


4 : 11
The mixture from 1
st
and 2
nd
container should be mixed in the ratio 4:11.

10 seconds for aptitude
140

A) Two solutions of 90% and 97% purity are mixed resulting in 21liters of mixture of 94% purity. How much
is the quantity of the first solution in the resulting mixture?


B) A mixture of 20ltrs of brandy and water contains 10% water. How much water should be added to it to
increase the percentage of water to 25%?

C) A cistern contains 50 liters of water. 5 litres of water is taken out of it adn replaced by wine. The process
is repeated again. Find the proportion of wine and water in the resulting mixture.
A mixture of milk and water measures 60 gallons. It contains 20% water. How many gallons of water
should be added to it so that water may be 25%?
A mixture of spirit and water measure 80 gallons. It contains 20% water. How much water should be
added to it so that water may be 25%?
There are two vessels A and B in which the ratio of milk and water are as 5:2 and 8:7 respectively.
Two gallons are drawn from vessel A and 3 gallons from vessel B, and are mixed in another empty
vessel. What is the ratio of milk and water in it?
Two gallons of mixture in which there is 2/5 of water and the rest spirit is mixed with five gallons of
mixture in which there is 1/3 of water and the rest spirit. What is the ratio of water and spirit in the
new mixture?
10 seconds for aptitude



One vessel contains a mixture of 5 parts pure wine and 3 parts soda, whereas the other vessel
contains a mixture of 9 parts pure wine and 5 parts soda. Compare the strength of the wine.
One milk can contains a mixture of milk and water in the ratio 7:5 and the other contains the mixture
of milk and water in which 2/5th is water. Compare their purity.
Rodrick mixes a martini that has a volume of 'n' ounces having 40% Vermouth and 60% Gin by
volume.
He wants to change it so that the martini is 25% Vermouth by volume. How many ounces of Gin must
he add?







10 seconds for aptitude
142

Data sufficiency
Each of the following problems contains a question followed by two statements, numbered (A) and (B). You
need not solve the problem; rather you must decide whether the information given is sufficient to solve the
problem.
The correct answer to a question is
A. if statement (A) ALONE is sufficient to answer the question but statement (B) alone is not sufficient;
B. if statement (B) ALONE is sufficient to answer the question but statement (1) alone is not sufficient;
C. if the two statements TAKEN TOGETHER are sufficient to answer the question, but NEITHER
statement ALONE is sufficient;
D. if EACH statement ALONE is sufficient to answer the question;
E. if the two statements TAKEN TOGETHER are still NOT sufficient to answer the question.
1. Quantity A: (-6)
4

Quantity B: (-6)
5

A. if the quantity A is greater;
B. if the quantity B is greater;
C. if the two quantities are equal;
D. if the relationship cannot be determined from the information given.
Ans : A
10 seconds for aptitude



2. Quantity A: Time to travel 95 miles at 50 miles per hour
Quantity B: Time to travel 125 miles at 60 miles per hour
A. Quantity A is greater
B. Quantity A equals Quantity B
C. Quantity B is greater
D. Relationship Indeterminate
Ans : C
3. Quantity A: (9/13)
2

Quantity B: (9/13)
1/2

A. Quantity A equals Quantity B
B. Relationship Indeterminate
C. Quantity B is greater
D. Quantity A is greater
Ans : C
4. Quantity A: 4 / 100
Quantity B: 0.012 / 3
A. Quantity B is greater
B. Quantity A equals Quantity B
C. Quantity A is greater
D. Relationship Indeterminate
10 seconds for aptitude
144

Ans : C
5. x = 2y + 3
y = -2
Quantity A: x
Quantity B: -1
A. if the quantity in Column A is greater
B. if the quantity in Column B is greater
C. if the two quantities are equal
D. if the relationship cannot be determined from the information given
Ans : C
6. x + 2y > 8
Quantity A: 2x + 4y
Quantity B: 20
A. if the quantity in Column A is greater
B. if the quantity in Column B is greater
C. if the two quantities are equal
D. if the relationship cannot be determined from the information given.
Ans : D
10 seconds for aptitude



7. Quantity A: The number of months in 7 years
Quantity B: The number of days in 12 weeks
A. if the quantity in Column A is greater
B. if the quantity in Column B is greater
C. if the two quantities are equal
D. if the relationship cannot be determined from the information given
Ans : C
8. Quantity A: 1-1/27
Quantity B: 8/9 + 1/81
A. if the quantity in is greater
B. if the quantity in is greater
C. if the two quantities are equal
D. if the relationship cannot be determined from the information given.
Ans : A
9. r/>s/>0/>
Quantity A: rs/r
Quantity B: rs/s
A. if the quantity A is greater
B. if the quantity B is greater
10 seconds for aptitude
146

C. if the two quantities are equal
D. if the relationship cannot be determined from the information given.
Ans : B
10. Quantity A: 0.83
Quantity B: 0.81/3
A. Quantity B is greater
B. Relationship Indeterminate
C. Quantity A is greater
D. Quantity A equals Quantity B
Ans : A
t is a positive integer.
4/7 = t/s
Quantity A: s
Quantity B:7
A. if the quantity in Column A is greater;
B. if the quantity in Column B is greater;
C. if the two quantities are equal;
D. if the relationship cannot be determined from the information given
Ans : D
10 seconds for aptitude




Quantity A: (0.82)
2
(0.82)
3

Quantity B:(0.82)
6

A. if the quantity in Column A is greater;
B. if the quantity in Column B is greater;
C. if the two quantities are equal;
D. if the relationship cannot be determined from the information given.
Ans : A
For all real numbers a, let a
*
= 1 - a.
Quantity A: ((-1)
*
)
*

Quantity B: 2
*

A. if the quantity in Column A is greater;
B. if the quantity in Column B is greater;
C. if the two quantities are equal;
D. if the relationship cannot be determined from the information given.
Ans : C
Quantity A: (x - 1)(x)(x + 1)
Quantity B:(x)(x)(x)
10 seconds for aptitude
148

A. if the quantity in Column A is greater;
B. if the quantity in Column B is greater;
C. if the two quantities are equal;
D. if the relationship cannot be determined from the information given.
Ans : D
Quantity A: (3 x 4 x 17) / (121 x 100)
Quantity B: (4 x 5 x 19) / (1000 x 121)
A. Quantity A is greater
B. Quantity A equals Quantity B
C. Relationship Indeterminate
D. Quantity B is greater
A
Consider a triangle PQR.
Quantity A: length of PQ + length of QR
Quantity B: length of PR
A. Quantity A is greater
B. Quantity B is greater
C. Relationship Indeterminate
D. Quantity A equals Quantity B
10 seconds for aptitude



Ans : A
Quantity A: (27 - 13) (296 + 534)
Quantity B: (27 + 13) (534 + 296)
A. Quantity B is greater
B. Quantity A equals Quantity B
C. Relationship Indeterminate
D. Quantity A is greater
Ans : A
Quantity A: A = 1.1
Quantity B: 12.1
1/2

A. Relationship Indeterminate
B. Quantity B is greater
C. Quantity A equals Quantity B
D. Quantity A is greater
Ans : B
100 < y < 200 and 100 < z < 210
Quantity A: y
Quantity B: z
10 seconds for aptitude
150

A. Quantity A is greater
B. Quantity A equals Quantity B
C. Quantity B is greater
D. Relationship Indeterminate
Ans : D
y
2
+ z
2
= 34 and yz = 15
Quantity A: y
2
+ 2yz + z
2

Quantity B: (y + z)
2

A. Quantity B is greater
B. Relationship Indeterminate
C. Quantity A is greater
D. Quantity A equals Quantity B


A data sufficiency question has a question followed by two statements. You will be expected to answer if the information
provided in the statements is sufficient to answer the question.

You answer will be
10 seconds for aptitude



1. Choice 1 if the question can be answered using one of the statements alone, while the other statement is not sufficient
to answer the question.
2. Choice 2 if the question can be answered using each of the statements independently
3. Choice 3 if both the statements together are needed to answer the question
4. Choice 4 if both the statements independently or taken together are not sufficient to answer the question
A collection of questions that typically appear from the topic of data sufficiency.
1. How long will it take for two pipes A and B to fill an empty cistern if they worked alternately for an hour each?

(A) Working alone, Pipe A can fill the cistern in 40 hours
(B) Pipe B is one third as efficient as Pipe A

What is the value of X, if X and Y are two distinct integers and their product is 30?

(A) X is an odd integer
(B) X > Y


2. The set S of numbers has the following properties:

I) If x is in S, then 1/x is in S.
II) If both x and y are in S, then so is x + y. Is 3 in S?

(A) 1/3 is in S.
(B) 1 is in S.
10 seconds for aptitude
152



3. Is x = y?

(A) = 4
(B) (x 50)
2
= (y 50)
2


4. Is the smallest of five consecutive integers even?

(A) The product of the five integers is 0
(B) The arithmetic mean of the five integers is 0.


5. Question 6: Is X a prime number, given that X is a positive integer?

(A) X
4
> 3000
(B) X
4
< 10,000


6. Question 7: Is m divisible by 6?

(A) m is divisible by 3
(B) m is divisible by 4
10 seconds for aptitude





7. Question 8: If a salesman received a commission of 3% of the sales that he has booked in a month, what was the
sales booked by the salesman in the month of November 2003?

Use the following answer choices for the questions below.
A. Statement 1 alone is sufficient but statement 2 alone is not sufficient to answer the question asked.
B. Statement 2 alone is sufficient but statement 1 alone is not sufficient to answer the question asked.
C. Both statements 1 and 2 together are sufficient to answer the question but neither statement is sufficient alone.
D. Each statement alone is sufficient to answer the question.
E. Statements 1 and 2 are not sufficient to answer the question asked and additional data is needed to answer the
statements.

1) If the average size of 3 accounts is $1 million, is the smallest account less than $500,000?
1. The largest account is $1.3 million.
2. One of the accounts is $0.7 million.
2) Is the product of x and y greater than 60?
1. The sum of x and y is greater than 60.
2. Each of the variables is greater than 2.
3) What is the value of y?
10 seconds for aptitude
154

1. y - 3 = 2
2. y
2
= 25
4) What was the percent increase of Company A's stock between June 1 and June 30, 2000?
1. The stock gained $5 in value during June 2000.
2. The stock rose 12% during the first half of the month.
5) Which company reported the larger dollar increase in earnings?
1. Company A reported that its earnings increased by 5%.
2. Company B reported that its earnings increased by 7%.
6) Is x a prime number?
1. x is an even number.
2. x can not be divided evenly by an odd number other than 1.




10 seconds for aptitude



Analytical Reasoning:
Analytical reasoning becomes easy when we have a clear cut method, good way of representing the
data
The method to solve questions using tables is described here.
Choosing the right table comes with practice and skill the following question is solved.

In a group of five persons A, B, C, D and E
a) A and C are intelligent in English and Reasoning.
b) B and C are intelligent in English and General Awareness.
c) E and D are intelligent in Arithmetic and Interview.
d) E is intelligent in Interview, reasoning and Arithmetic.
c) E and D are intelligent in Arithmetic and Interview.

Arithmetic English Interview General
Awareness
reasoning
A
B
C
D
E
10 seconds for aptitude
156


e)B and D are intelligent in Arithmetic and General Awareness.

1) Who is intelligent in English, arithmetic and general
awareness (ans:B)
2) Who is intelligent in english and reasoning but not in
general awareness (ans: A)
3) Who is intelligent in Arithmetic, General Awareness and
Interview (ans:D)
4) Who is intelligent in English General Awareness and Reasoning
(ans:C)
5) Who is intelligent in Arithmetic, Reasoning and Interview(ans:E)

The answers become obvious once the table is complete.

There are six steps that lead from the first to the second floor. No two people can be on the same step Mr.
A is two steps below Mr. C Mr. B is a step next to Mr. D Only one step is vacant ( No one standing on that
step ) Denote the first step by step 1 and second step by step 2 etc.

10 seconds for aptitude



1. If Mr. A is on the first step, Which of the following is true?
(a) Mr. B is on the second step
(b) Mr. C is on the fourth step.
(c) A person Mr. E, could be on the third step
(d) Mr. D is on higher step than Mr. C.

Ans: (d)

2. If Mr. E was on the third step & Mr. B was on a higher step than Mr. E which step must be vacant
(a) step 1
(b) step 2
(c) step 4
(d) step 5
(e) step 6

Ans: (a)

3. If Mr. B was on step 1, which step could A be on?
(a) 2&e only
(b) 3&5 only
10 seconds for aptitude
158

(c) 3&4 only
(d) 4&5 only
(e) 2&4 only

Ans: (c)


4. If there were two steps between the step that A was standing and the step that B was standing on, and
A was on a higher step than D , A must be on step
(a) 2
(b) 3
(c) 4
(d) 5
(e) 6

Ans: (c)


5. Which of the following is false
i. B&D can be both on odd-numbered steps in one configuration
10 seconds for aptitude



ii. In a particular configuration A and C must either both an
odd numbered steps or both an even-numbered steps
iii. A person E can be on a step next to the vacant step.
(a) i only
(b) ii only
(c) iii only
(d) both i and iii

Ans: (c)

Directions for questions 6-9: The questions are based on the information given below

Six swimmers A, B, C, D, E, F compete in a race. The outcome is as follows.
i. B does not win.
ii. Only two swimmers separate E & D
iii. A is behind D & E
iv. B is ahead of E , with one swimmer intervening
v. F is a head of D

6. Who stood fifth in the race ?
10 seconds for aptitude
160

(a) A
(b) B
(c) C
(d) D
(e) E

Ans: (e)


7. How many swimmers seperate A and F ?
(a) 1
(b) 2
(c) 3
(d) 4
(e) cannot be determined

Ans: (d)


8. The swimmer between C & E is
10 seconds for aptitude



(a) none
(b) F
(c) D
(d) B
(e) A

Ans: (a)


9. If the end of the race, swimmer D is disqualified by the Judges then swimmer B finishes in which place
(a) 1
(b) 2
(c) 3
(d) 4
(e) 5

Ans: (b)


Directions for questions 10-14: The questions are based on the information given below:-
10 seconds for aptitude
162


Five houses lettered A,B,C,D, & E are built in a row next to each other. The houses are lined up in the
order A,B,C,D, & E. Each of the five houses has a colored chimney. The roof and chimney of each
housemust be painted as follows.
i. The roof must be painted either green,red ,or yellow.
ii. The chimney must be painted either white, black, or red.
iii. No house may have the same color chimney as the color of roof.
iv. No house may use any of the same colors that the every next house uses.
v. House E has a green roof.
vi. House B has a red roof and a black chimney

10. Which of the following is true ?
(a) At least two houses have black chimney.
(b) At least two houses have red roofs.
(c) At least two houses have white chimneys
(d) At least two houses have green roofs
(e) At least two houses have yellow roofs

Ans: (c)

10 seconds for aptitude




11. Which must be false ?
(a) House A has a yellow roof
(b) House A & C have different color chimney
(c) House D has a black chimney
(d) House E has a white chimney
(e) House B&D have the same color roof.

Ans: (b)


12. If house C has a yellow roof. Which must be true.
(a) House E has a white chimney
(b) House E has a black chimney
(c) House E has a red chimney
(d) House D has a red chimney
(e) House C has a black chimney

Ans: (a)

10 seconds for aptitude
164


13. Which possible combinations of roof & chimney can house
I. A red roof 7 a black chimney
II. A yellow roof & a red chimney
III. A yellow roof & a black chimney

(a) I only
(b) II only
(c) III only
(d) I & II only
(e) I&II&III

Ans: (e)


14. What is the maximum total number of green roofs for houses
(a) 1
(b) 2
(c) 3
(d) 4
10 seconds for aptitude



(e) 5

Miss Dean wants to rennovate her house. She hires a plumber, a carpenter, a painter, an electrician and
an interior decorator. The work to be finished in one working (Monday - Friday ). Each worker will take
the full day to do his job. Miss Dean permits only one person to work each day.
I. The painter can work only after the plumber and the carpenter have finished their jobs
II. The interior decorator must do his job before the electrician.
III. The carpenter cannot work on Monday or Tuesday


15. If the painter work on Thursday, which one of the following alternatives is possible?
(a) The electrician works on Tuesday.
(b). The electrician works on Friday.
(c) The interior decorator works after the painter does.
(d). The painter works on consecutive days.
(e). Miss Dean cannot fit all of the workers int schedule

Ans: (b)


10 seconds for aptitude
166

16. If the painter works on Friday which of the following must be false?
(a) . The carpenter may works on Wednesday
(b). The carpenter and the electrician may work on consecutive days
(c). If the carpenter works on Thursday, the electrician has to work on Wednesday
(d). The plumber may work before the electrician does
(e). The electrician may work on Tuesday

Ans: (c)


17. Which argument is possible?
(a). The electrician will works on Tuesday and the interior decorator on Friday
(b). The painter will work on wednesday and plumber on thursday
(c). The carpenter will works on Tuesday and the painter on Friday
(d). THe painter will work on Monday and the carpenter on Thursday
(e). The carpenter will work on Wednesday and the plumber on Thursday

Ans: (e)

10 seconds for aptitude

Potrebbero piacerti anche